Примеры на вычитание 4 класс в столбик: 4 класс, задачи на вычитание многозначных чисел и дробей, столбиком

Содержание

Урок 20. вычитание с переходом через несколько разрядов вида 30 007 – 648 — Математика — 4 класс

Математика, 4 класс

Урок № 20. Вычитание с переходом через несколько

разрядов вида 30 007 — 648

Перечень вопросов, рассматриваемых в теме:

1 .Что такое десятичная система счисления?

2. Как надо выполнять вычитание многозначных чисел с переходом через несколько разрядов?

3. Что ставим при записи вычитания в столбик над разрядами, которые подвергались дроблению?

Глоссарий по теме:

Многозначные числа – это числа класса тысяч и класса миллионов.

Класс единиц – единицы, десятки, сотни. Это – первый класс.

Класс тысяч – единицы тысяч, десятки тысяч, сотни тысяч. Это – второй класс. Единица этого класса – тысяча.

Класс миллионов – единицы миллионов, десятки миллионов, сотни миллионов. Это – третий класс. Единица этого класса – миллион.

Обязательная литературы и дополнительная литература:

  1. Моро М. И., Бантова М. А. и др. Математика 4 класс. Учебник для общеоразовательных организаций М.; Просвещение, 2014. – С. 61

2. Рудницкая В. Н., Юдачева Т. В. Математика: 4 класс: учебник в 2 ч. Ч.1. – М. Вентана-Граф, 2017. – С. 13-15

  1. Готовимся к Всероссийской проверочной работе. Математика. Рабочая тетрадь 4 класс/ под. ред. Г. С. Ковалевой. – М.; Просвещение, 2017. – С. 6 — 7

Теоретический материал для самостоятельного изучения

Вспомним десятичную систему счисления:

1 тыс. = 10 сот.

1 сот. = 10 дес.

1 дес. = 10 ед.

1 млн. = 10 сот. тыс.

1 сот. тыс. = 10 дес. тыс.

1 дес. тыс. = 10 ед. тыс.

1 дес. – 1 (Чтобы из 1 дес. вычесть единицу, заменим 1 дес. десятью единицами и вычтем 1из 10, получится 9)

1 сот. – 1 дес. ( изз 1 сот. вычесть 1 дес., заменим 1 сот 10 дес. и вычтем 1 дес. из 10, останется 9 дес., или 90)

1 тыс. – 1 сот. ( из 1 тыс. вычитаем 1 сотню, заменим 1 тыс. 10 сотнями и вычтем 1 сотню из 10, останется 9 сотен или 900)

Рассмотрим приём письменного вычисления (в столбик), из круглых чисел, используя занимание единицы через несколько разрядов. Запишите единицы под единицами, десятки под десятками.

Вычисляем десятки. Мы добавили к десяткам одну сотню, то есть десять десятков, но при этом заняли десяток, осталось 9 дес. Чтобы не забыть запишем наверху в разряде десятков цифру 9. 9 – 2 = 7

Вычисляем сотни. Мы заняли одну сотню из шести, поэтому записываем пять сотен. Итого результат вычисления равнее 574.

А теперь попробуем рассмотреть особый случай вычитания: когда в записи уменьшаемого есть несколько нулей. Обнаружилась проблема, предыдущего алгоритма вычисления не хватает, т. к. приходится занимать единицу через несколько разрядов и выполнять раздробление в единицы низших разрядов не один раз. Запишем единицы под единицами, десятки под десятками, сотни под сотнями.

Из семи вычесть восемь нельзя, потому занимаем один десяток. Но единицы в разряде десятков отсутствуют, потому занимаем сотню. Отсутствуют сотни и отсутствуют единицы тысяч, потому занимаем один десяток тысяч.

Один десяток тысяч – это десять единиц тысяч, можем занять одну тысячу. Одна тысяча – это 10 сотен, можем занять из сотен. Одна сотня – это десять десятков, теперь занимаем один десяток. Один десяток – это десять единиц да ещё 7. 17 – 8 = 9.

Вычисляем десятки. Мы добавили 10 десятков, но один заняли, чтобы не забыть, запишем наверху в разряде десятков цифру 9. 9 – 4 = 5 десятков.

Вычисляем сотни. Мы добавили 10 сотен, но одну заняли, осталось 9. Чтобы не забыть, запишем наверху в разряде сотен цифру 9. 9 – 6 = 3 сотни.

Вычисляем единицы тысяч. Мы добавили 10 единиц тысяч, но одну заняли, осталось 9 единиц тысяч. Чтобы не забыть, запишем наверху в разряде единиц тысяч цифру 9. Переписываем 9, так как единицы тысяч в вычитаемом отсутствуют.

Осталось два десятка тысяч, их переписываем. Итого результат вычисления равен 29 359.

Вывод: Ответим на вопросы, поставленные в начале урока. Десятичная система счисления – позиционная система счисления по основанию 10. Вычитание многозначных чисел с переходом через несколько разрядов надо выполнять поразрядно: из единиц – единицы, из десятков – десятки, из сотен – сотни, из тысяч – тысячи. При записи вычитания в столбик для памяти ставим точки над теми разрядами, которые подвергались дроблению (замена 1 единицы высшего разряда 10 единицами низшего соседнего разряда.).

Выполним несколько тренировочных заданий.

1. К каждой позиции первого столбика подберите соответствующую позицию второго:

Варианты ответов:

300 025 – 147 328

424 399

268 765 + 183 426

152 697

507 425 – 83 026

127 507

506 021 – 378 514

452 191

Правильный вариант/варианты:

300 025 – 147 328

152 697

268 765 + 183 426

452 191

507 425 – 83 026

424 399

506 021 – 378 514

  1. 7

Выполните вычисления:

Правильный вариант:

  1. 7 258 – 3 028 = 4 230
  2. 20 456 + 5703 = 26 159
  3. 93 698 – 542 = 93 156
  4. 121 212 + 434 343 = 555 555

Урок.

«Сложение и вычитание многозначных чисел» 4 класс. Школа России

МБОУ   Новоселковская   СШ

 

 

 

 

 

   

 

 

 

 

 

 

Подготовила и провела: Чернышова Н.В.,

                                                учитель начальных классов

                                                                 высшей квалификационной категории

 

 

 

 

 

 

15 ноября, 2017 г.

 

 

Тема урока: Сложение и вычитание  многозначных чисел.

УМК: Школа России.

Тип урока: урок комплексного применения знаний и умений (урок изучения новой темы).

Цель урока: Отработка вычислительных навыков с многозначными числами.

Планируемый результат обучения, в том числе и формирование УУД:

— учащиеся научатся складывать и вычитать многозначные числа, систематизируют и закрепят знания, полученные на уроках математики.

Формируемые УУД:

Познавательные:

—     формировать умения самостоятельно выделять и формулировать познавательную цель всего урока и отдельного задания;

—     совершенствовать умение детей выполнять действия с многозначными числами;

—     отрабатывать навыки решения задач изученных видов;

—     строить логическое рассуждение.

Коммуникативные:

—     развивать умение грамотно, логично, полно давать ответы, уметь доказать, аргументировать свое мнение;

—     формировать умение работать в паре, группе,  находить общее решение, умение аргументировать своё предложение;

—     развивать способность сохранять доброжелательное отношение друг к другу, взаимоконтроль и взаимопомощь по ходу выполнения задания;

—     Регулятивные:

—     создать благоприятный психологический климат для возможного раскрытия потенциала каждого ребенка;

—     проявлять познавательную инициативу в учебном сотрудничестве.

Личностные:

—     развивать умение применять свои знания в ситуации, близкой к жизненной;

—     формировать способности к самооценке на основе критериев успешности учебной деятельности.

 

Оборудование и материалы: учебник для 4-го  класса «Математика» // Моро М.И., Бантова М.А., Бельтюкова Г.В. и др. – М.: Просвещение, 2015; компьютер, проектор; карточки математического лото, карточки быстрого счета, карточки с задачами, «Лист оценки работоспособности на уроке», конверты с домашним заданием.

 

 

 

 

 

 

 

Ход урока

 

1.Организационный момент. Самоопределение к деятельности.

 

— Добрый день!

— Ребята, сегодня на уроке мы продолжим постигать тайны царицы всех наук – Математики. Нашими помощниками будут: знания, умения, старание.

 

      2.  Актуализация знаний и фиксация затруднения в деятельности

1. -Проверим вашу готовность к предстоящей работе.

I. Что это такое?

o   как называются числа при сложении?

o   как называются числа при вычитании?

o   как узнать на сколько одно число больше или меньше другого?

o   на какие группы можно разделить числа?

o   как узнать во сколько раз одно число больше или меньше другого?

o   что такое периметр?

o   как найти площадь?

o   что такое квадрат?

o   как найти неизвестное слагаемое?

o   Как проверить вычитание?

o   Как проверить сложение?

o   если к числу прибавить 0, то…(то число не изменится)?

o   если из числа вычесть 0, то…(число не изменится)?

o   разность двух одинаковых чисел равна…(0)

o   какое число получится при делении на 0?

o   сколько дней в году?

o   сколько кг в 1 т.?

o   сколько ушей в классе у присутствующих детей? (шутка)

II. Вычисли наиболее лёгким способом.

96+98+904 =                                

61+58+39=

24+65+76=                        

2. Индивидуальная работа  (карточки).

a — b : c + o

(a + o c) : b . k

a + o – c : b

(100 . 9 :30) +120 :2 + 140

– Откройте свои тетради. Запишем

15 ноября.

Классная работа.

 

 

3. Игра «Математическое лото».

У вас карточки с числами, я задаю вам выражение, вы находите ответ и зачёркиваете:

1.    Найдите число, в котором 3 тысячи, 2 сотни, 5 десятков, 4 единицы. (3254)

2.    Найдите одну пятую числа 600.  (120)

3.    Увеличь в 4 раза число 250 .   (1000)

4.   

Уменьшаемое 320 вычитаемое 30.Найдите частное. (290)

5.    На стадионе 5000 мест. Половина из них, заняты зрителями. Сколько мест свободных? (2500)

6.    9 сотен умножить на 7.  (6300)

7.    7 часов — сколько это минут. (420)

8.    На сколько надо увеличить 25, чтобы получить 170? (145)

9.    Найдите площадь прямоугольника со сторонами 700 м и 4 м (2800)

10.41 увеличить в 100 раз. (4100)

11.32 центнера – сколько кг?(3200)

 

 

 

120

145

3200

2500

290

32496

3254

854671

420

6300

4100

2800

1000


Взаимопрверка  (меняются тетрадями)

Оцените работу товарища.

— На какие две группы можно разделить эти числа (чётные и нечётные), (трёхзначные и четырёхзначные).

— Какие числа вы не зачеркнули?

4. Запишите числа из таблицы, которые вы не зачеркнули, в порядке возрастания. (один на доске)

Проверка! 32496, 854671

-Дайте характеристику любому из этих числу ОЦЕНКА!

Какие арифметические действия, вы можете выполнить с этими числами? (сложение и вычитание)

5.Постановка учебной задачи.

— Как вы, думаете, над какой темой будем работать сегодня?

  • Итак, тема урока: Сложение и вычитание многозначных чисел.
  • Какую задачу перед собой поставим?

Научиться письменному сложению и вычитанию многозначных чисел.

6.Построение проекта выхода из затруднения, открытие нового знания

— Чтобы правильно решать такие примеры, нужно вспомнить  алгоритм решения.

— Что такое алгоритм? (План)

— Сейчас вы самостоятельно вспомните алгоритм сложения и вычитания 3-х значных чисел и  попробуете его восстановить. 

У вас на партах карточки, на которых напечатаны разные действия алгоритма. Работая и обсуждая в парах, расставьте порядок  алгоритма  для сложения и вычитания многозначных чисел.

I вариант

Алгоритм письменного сложения многозначных чисел

Для того чтобы выполнить сложение многозначных чисел, надо:

1.      Пишу единицы под единицами, десятки под десятками, сотни под сотнями, единицы тысяч под единицами тысяч, десятки тысяч под десятками тысяч, сотни тысяч под сотнями тысяч.

2.      Складываю единицы …….

3.      Складываю десятки …….

4.      Складываю сотни ……

5.      Складываю единицы тысяч

6.      Складываю десятки тысяч

7.      Складываю сотни тысяч

8.      Читаю ответ …….

II вариант

Алгоритм письменного вычитания многозначных чисел

Для того чтобы выполнить вычитание многозначных чисел,
надо:

  1. Пишу единицы под единицами, десятки под десятками, сотни под сотнями, единицы тысяч под единицами тысяч, десятки тысяч под десятками тысяч, сотни тысяч под сотнями тысяч.
  2. Вычитаю единицы …….
  3. Вычитаю десятки …….
  4. Вычитаю сотни ……
  5. Вычитаю единицы тысяч
  6. Вычитаю десятки тысяч
  7. Вычитаю сотни тысяч
  8. Читаю ответ …….

 

ПРОВЕРКА (дети зачитывают алгоритм сложения и вычитания) ОЦЕНКА!

 

Что самое главное при решении?

(правильно записывать числа, чтобы единицы разрядов совпадали, вычислять начинать с единиц)

+ ESAKHP             + ESAKHP          + ESAKHP     

   LVFA                        LVFA                     LVFA

 

 

ФИЗКУЛЬТМИНУТКА

 

1.)- Составьте из чисел, которые вы выписали, все возможные варианты примеров (дети называют устно)

Решим у доски, применяя алгоритм сложения и вычитания многозначных чисел.

Решение примеров на доске.  «Поспешишь – людей насмешишь»

  854671+32496 =

  854671-32496 =

 

2. ) Первичное закрепление.

 

«Все тайное становится явным»

 (решение примеров по вариантам с взаимопроверкой)

 

У вас на партах лежат листы с заданием. Каждый из вас сам выбирает тот вариант заданий, с которым он справится.

Вариант 1

98765+2232 =100.997            189456-23124=166.332

Вариант 2

3806578+344612=4.151.190            7689438-828129=6.861.309

Вариант 3

3274560198+64789321= 3339349519     6472318059-708913724=3339349519

 Самостоятельно решаем  и выполняем  проверку по слайду.

 

 

3.) Творческое применение и добывание знаний в новой ситуации.

 

«Без труда нет добра» (решение задач, самостоятельная работа)

 

Откроем учебник с. 60  Задача 264.

 

Выдано -327 к.

Осталось -? на 246 r <

1)    _327

  246

              81 (к. ) –осталось

2)    _З27

    81

  408 (к.) – всего книг

Ответ: 408 книг.

Дополнительно : № 265

 

1.Укажи верно решённый пример:

 

   518.304              518.304            518.304                 

+ 420.296          + 420.296          +420.296              

   939.600              938.600            939.590                 

 

2.  Укажи верно решённый пример:

 

 _  54.762            _ 54.762               _54.762                 

     49.826               49.826                 49.826                   

       4.934                 5.946                   4.936                    

 

 

1.Запиши вычисления в столбик:

26 637 + 176                                  12 765 – 2937

34 600 + 2548                                63 243 — 62 754

428 095 + 74 340                          176850 – 94768

120 470 + 639 842                        535 200 – 484940

 

 

 

 

Подведение итогов

 

6. Домашнее задание: с.60 № 262, 263.

 

7. Рефлексия.

Тихо все на месте сядем

И закроем глазки.

Вспомним, всё, что повторяли,

Без моей подсказки.

 

— В чём вы упражнялись на уроке?

— Ребята, оцените свою работоспособность на уроке.

Работал (ла) на уроке:

а) отлично (смайлик с улыбкой),

б) хорошо (смайлик с удивлением),

в) мог (ла) бы работать лучше (смайлик грустный).

 

Спасибо за урок!

 

 

 

 

 

 

 

 

 

 

 

 

 

 

 

 

 

 

 

 

a — b : c + o

(a + o — c) : b . k

a + o – c : b

(100 . 9 :30) +120 :2 + 140

 

a — b : c + o

(a + o — c) : b . k

a + o – c : b

(100 . 9 :30) +120 :2 + 140

 

a — b : c + o

(a + o — c) : b . k

a + o – c : b

(100 . 9 :30) +120 :2 + 140

 

a — b : c + o

(a + o — c) : b . k

a + o – c : b

(100 . 9 :30) +120 :2 + 140

 

a — b : c + o

(a + o — c) : b . k

a + o – c : b

(100 . 9 :30) +120 :2 + 140

 

a — b : c + o

(a + o — c) : b . k

a + o – c : b

(100 . 9 :30) +120 :2 + 140

 

 

 

 

 

 

 

 

 

Вариант 1

98765+2232 =                              189456-23124=

Вариант 2

3806578+344612=                       7689438-828129=

Вариант 3

3274560198+64789321=              6472318059-708913724=

 

Вариант 1

98765+2232 =                              189456-23124=

Вариант 2

3806578+344612=                       7689438-828129=

Вариант 3

3274560198+64789321=              6472318059-708913724=

 

Вариант 1

98765+2232 =                              189456-23124=

Вариант 2

3806578+344612=                       7689438-828129=

Вариант 3

3274560198+64789321=              6472318059-708913724=

 

Вариант 1

98765+2232 =                              189456-23124=

Вариант 2

3806578+344612=                       7689438-828129=

Вариант 3

3274560198+64789321=              6472318059-708913724=

Алгоритм письменного сложения многозначных чисел

  • Складываю единицы …….
  • Складываю единицы тысяч
  • Пишу единицы под единицами, десятки под десятками, сотни под сотнями, единицы тысяч под единицами тысяч, десятки тысяч под десятками тысяч, сотни тысяч под сотнями тысяч……
  • Складываю сотни тысяч
  • Читаю ответ …….
  • Складываю десятки …….
  • Складываю сотни ……
  • Складываю десятки тысяч

 

Алгоритм письменного вычитания многозначных чисел

  • Вычитаю единицы …….
  • Читаю ответ …….
  • Вычитаю сотни ……
  • Вычитаю десятки тысяч
  • Пишу единицы под единицами, десятки под десятками, сотни под сотнями, единицы тысяч под единицами тысяч, десятки тысяч под десятками тысяч, сотни тысяч под сотнями тысяч.
  • Вычитаю десятки …….
  • Вычитаю единицы тысяч
  • Вычитаю сотни тысяч

 

 

 

 

 

 

120

145

3200

2500

290

32496

3254

854671

420

6300

4100

2800

1000

 

 

 

 

 

120

145

3200

2500

290

32496

3254

854671

420

6300

4100

2800

1000

 

 

 

 

 

120

145

3200

2500

 

290

32496

3254

854671

420

 

6300

4100

2800

1000

 

1. Укажи верно решённые  примеры:

 

   518.304              518.304            518.304                 

+ 420.296          + 420.296          +420.296              

   939.600              938.600            939.590                 

 

 _  54.762            _ 54.762               _54.762                 

     49.826               49.826                 49.826                   

       4.934                 5.946                   4.936 

                  

1.Укажи верно решённые  примеры:

 

   518.304              518.304            518.304                 

+ 420.296          + 420.296          +420.296               

   939.600              938. 600            939.590                 

 

 _  54.762            _ 54.762               _54.762                 

     49.826               49.826                 49.826                   

       4.934                 5.946                   4.936                    

1.Укажи верно решённые  примеры:

 

   518.304              518.304            518.304                 

+ 420.296          + 420.296          +420.296              

   939.600              938.600            939.590                 

 

 _  54.762            _ 54.762               _54.762                 

     49.826               49.826                 49.826                   

       4. 934                 5.946                   4.936 

                  

1.Укажи верно решённые  примеры:

 

   518.304              518.304            518.304                 

+ 420.296          + 420.296          +420.296              

   939.600              938.600            939.590                 

 

 _  54.762            _ 54.762               _54.762                 

     49.826               49.826                 49.826                   

       4.934                 5.946                   4.936                    

1.Укажи верно решённые  примеры:

 

   518.304              518.304            518.304                 

+ 420. 296          + 420.296          +420.296              

   939.600              938.600            939.590                 

 

 _  54.762            _ 54.762               _54.762                 

     49.826               49.826                 49.826                   

       4.934                 5.946                   4.936 

                  

1.Укажи верно решённые  примеры:

 

   518.304              518.304            518.304                 

+ 420.296          + 420.296          +420.296              

   939.600              938.600            939.590                 

 

 _  54.762            _ 54.762               _54.762                 

     49. 826               49.826                 49.826                   

       4.934                 5.946                   4.936                    

1.Укажи верно решённые  примеры:

 

   518.304              518.304            518.304                 

+ 420.296          + 420.296          +420.296               

   939.600              938.600            939.590                 

 

 _  54.762            _ 54.762               _54.762                 

     49.826               49.826                 49.826                   

       4.934                 5.946                   4.936

                  

1.Укажи верно решённые  примеры:

 

   518. 304              518.304            518.304                 

+ 420.296          + 420.296          +420.296              

   939.600              938.600            939.590                 

 

 _  54.762            _ 54.762               _54.762                 

     49.826               49.826                 49.826                   

       4.934                 5.946                   4.936                    

 

 

 

 

 

 

 

 

1.Запиши вычисления в столбик:

26 637 + 176                                  12 765 – 2937

34 600 + 2548                                63 243 — 62 754

428 095 + 74 340                          176850 – 94768

120 470 + 639 842                        535 200 – 484940

 

1. Запиши вычисления в столбик:

26 637 + 176                                  12 765 – 2937

34 600 + 2548                                63 243 — 62 754

428 095 + 74 340                          176850 – 94768

120 470 + 639 842                        535 200 – 484940

 

 

1.Запиши вычисления в столбик:

26 637 + 176                                  12 765 – 2937

34 600 + 2548                                63 243 — 62 754

428 095 + 74 340                          176850 – 94768

120 470 + 639 842                        535 200 – 484940

 

1.Запиши вычисления в столбик:

26 637 + 176                                  12 765 – 2937

34 600 + 2548                                63 243 — 62 754

428 095 + 74 340                          176850 – 94768

120 470 + 639 842                        535 200 – 484940

 

ГДЗ по математике 4 класс учебник Моро, Бантова 1 часть

❤️️Ответ к странице 60. Математика 4 класс учебник 1 часть. Авторы: М.И. Моро, М.А. Бантова.

Номер 260.

(Устно.) Вычисли наиболее лёгким способом.

Ответ:

2 + 96 + 98 + 904 = (2 + 98) + (96 + 904) = 100 + 1000 = 1100 257 + 18 + 12 + 3 + 40 = (257 + 3) + (18 + 12) + 40 = 260 + 30 + 40 = (260 + 40) + 30 = 300 + 30 = = 330 48 + 530 + 70 + 52 = (48 + 52) + (530 + 70) = 100 + 600 = 700 33 + 34 + 35 + 36 + 37 =(33 + 37) + (34 + 36) + 35 =70 + 70 + 35 = 140 + 35 = 175

Объясни, как выполнено сложение и вычитание.

Ответ:

При записи столбиком сложения и вычитания чисел с большим количеством разрядов пользуются правилом: единицы записываются под единицами, десятки под десятками, сотни под сотнями, единицы тысяч под единицами тысяч и так далее.
Первый пример: Записываем разряды под разрядами и приступаем к сложению. Складываем единицы: шесть плюс два будет восемь. Складываем десятки: два плюс девять будет одиннадцать. Пишем единицу, а 1 десяток запоминаем. Складываем сотни: один плюс два будет три, да еще один-четыре. Складываем единицы тысяч: три плюс четыре получится 7. Читаем ответ: семь тысяч четыреста восемнадцать.
Второй пример: Записываем разряды под разрядами. Видим, что данный пример на вычитание. Приступаем к решению. Вычитаем единицы: от шести семь не отнимешь, значит, занимаем десяток. Шестнадцать минус семь будет девять. Вычитаем десятки: было четыре стало три, потому что заняли десяток. Три минус ноль будет три. Вычитаем сотни: от трех четыре не отнимешь, значит, занимаем десяток. Тринадцать минус четыре будет девять. Вычитаем единицы тысяч: было 5, занимали единицу, осталось 4. От 4 отнять 3 получится 1. Сносим десятки тысяч в ответ. Читаем ответ: двадцать одна тысяча девятьсот тридцать девять.

Номер 261.

Объясни, что означают записи в рамках на полях.

Ответ:

1) b + 0 = b, если к любому числу прибавить 0, то получится это же число. 2) 0 + c = c, если к нулю прибавить любое число, то получится прибавленное число. 3) a – 0 = a, если из любого числа вычесть 0, то получится это же число. 4) k – k = 0, если из любого числа вычесть себя же, то получится 0.

Номер 262.

Вычисли, записывая решение столбиком, и проверь сложение вычитанием, а вычитание сложением.

Ответ:

Номер 263.

В соревнования участвовало 18 семей, состоящих из 3 человек, и 16 семей, состоящих из 4 человек. Сколько это всего человек?

Ответ:


1) 3 ∙ 18 = 54 (чел.) – в 18 семьях. 2) 16 ∙ 4 = 64 (чел.) – в 16 семьях. 3) 54 + 64 = 118 (ч.) 3 ∙ 18 + 16 ∙ 4 = 118 (ч.) Ответ: 118 человек всего.

Номер 264.

После того как школьникам было выдано 327 книг, в библиотеке осталось на 246 книг меньше, чем было выдано. Сколько всего книг в библиотеке?

Ответ:


1) 327 − 246 = 81 (кн.) – осталось. 2) 327 + 81 = 408 (кн.) Ответ: 408 книг было в библиотеке.

Номер 265.

Составь задачу по выражению: 100 − (68 + 14)

Ответ:

На склад привезли 100 игрушек, из них 14 кукол и 68 мячей, а остальные игрушки были машинки. Сколько было машинок?
Всего – 100 шт. Куклы – 14 шт. Мячи – 68 шт. Машинки – ? шт. 100 − (68 + 14) = 18 (шт.) Ответ: 18 машинок.

Номер 266.

(Устно.)

Ответ:

999 + 1 = 1000 1000 − 1 = 999
10000 − 1 = 9999 9999 + 1 = 10000
50000 − 1 = 49999 80000 − 1 = 79999

Задание внизу страницы

Вычисли.

Ответ:

1050 − 50 = 1000 20000 + 800 = 20800 35840 − 840 = 35000

Задание на полях страницы

Ребус.

Ответ:

394 − 285 = 109

Рейтинг

Выберите другую страницу

1 часть

Учебник Моро3456789101112131415161718192021222324252627282930313233343536373839404142434445464748495051525354555657585960616263646566676869707172737475767778798081828384858687888990919293949596979899100101102103104105106107108109110111

2 часть

456789101112131415161718192021222324252627282930313233343536373839404142434445464748495051525354555657585960616263646566676869707172737475767778798081828384858687888990919293949596979899100101102103104105106107108109110111112113114115116117118119120121122123124125126127

Решение столбиком сложение и вычитание.

Как правильно объяснить ребёнку деление в столбик

Деление столбиком или, правильнее сказать, письменный прием деления уголком, школьники проходят уже в третьем классе начальной школы, но зачастую этой теме уделяется так мало внимания, что к 9-11 классу не все ученики могут им свободно пользоваться. Деление столбиком на двузначное число проходят в 4 классе, как и деление на трехзначное число, а далее этот прием используется только как вспомогательный при решении каких-либо уравнений или нахождении значения выражения.

Очевидно, что уделив делению столбиком больше внимания, чем заложено в школьной программе, ребенок облегчит себе выполнение заданий по математике вплоть до 11 класса. А для этого нужно немногое — понять тему и позаниматься, порешать, держа алгоритм в голове, довести навык вычисления до автоматизма.

Алгоритм деления столбиком на двузначное число

Как и при делении на однозначное число, будем последовательно переходить от деления более крупных счетных единиц к делению более мелких единиц.

1. Находим первое неполное делимое . Это число, которое делится на делитель с получением числа больше или равного 1. Это значит, что первое неполное делимое всегда больше делителя. При делении на двузначное число в первом неполном делимом минимум 2 знака.

Примеры 76 8:24. Первое неполное делимое 76
265 :53 26 меньше 53, значит не подходит. Нужно добавить следующую цифру (5). Первое неполное делимое 265.

2. Определяем количество цифр в частном . Для определения числа цифр в частном следует помнить, что неполному делимому соответствует одна цифра частного, а всем остальным цифрам делимого — еще по одной цифре частного.

Примеры 768:24. Первое неполное делимое 76. Ему соответствует 1 цифра частного. После первого неполного делителя есть еще одна цифра. Значит в частном будет всего 2 цифры.
265:53. Первое неполное делимое 265. Оно даст 1 цифру частного. Больше в делимом цифр нет. Значит в частном будет всего 1 цифра.
15344:56. Первое неполное делимое 153, а после него еще 2 цифры. Значит в частном будет всего 3 цифры.

3. Находим цифры в каждом разряде частного . Сначала найдем первую цифру частного. Подбираем такое целое число, чтобы при умножении его на наш делитель получилось число, максимально приближенное к первому неполному делимому. Цифру частного записываем под уголок, а значение произведения вычитаем столбиком из неполного делителя. Записываем остаток. Проверяем, что он меньше делителя.

Затем находим вторую цифру частного. Переписываем в строку с остатком цифру, следующую за первым неполным делителем в делимом. Полученное неполное делимое снова делим на делитель и так находим каждое последующее число частного, пока не закончатся цифры делителя.

4. Находим остаток (если есть).

Если цифры частного закончились и получился остаток 0, то деление выполнено без остатка. В ином случае значение частного записывается с остатком.

Так же выполняется деление на любое многозначное число (трехзначное, четырехзначное и т. д.)

Разбор примеров на деление столбиком на двузначное число

Сначала рассмотрим простые случаи деления, когда в частном получается однозначное число.

Найдем значение частного чисел 265 и 53.

Первое неполное делимое 265. Больше в делимом цифр нет. Значит в частном будет однозначное число.

Чтобы было легче подобрать цифру частного, разделим 265 не на 53, а на близкое круглое число 50. Для этого 265 разделим на 10, будет 26 (остаток 5). И 26 разделим на 5, будет 5 (остаток 1). Цифру 5 нельзя сразу записывать в частном, поскольку это пробная цифра. Сначала нужно проверить, подойдет ли она. Умножим 53*5=265. Мы видим, что цифра 5 подошла. И теперь можем ее записать в частном под уголок. 265-265=0. Деление выполнено без остатка.

Значение частного чисел 265 и 53 равно 5.

Иногда при делении пробная цифра частного не подходит, и тогда ее нужно менять.

Найдем значение частного чисел 184 и 23.

В частном будет однозначное число.

Чтобы было легче подобрать цифру частного, разделим 184 не на 23, а на 20. Для этого разделим 184 на 10, будет 18 (остаток 4). И 18 разделим на 2, будет 9. 9 – это пробная цифра, мы ее сразу писать в частном не будем, а проверим, подойдет ли она. Умножим 23*9=207. 207 больше, чем 184. Мы видим, что цифра 9 не подходит. В частном будет меньше 9. Попробуем, подойдет ли цифра 8. Умножим 23*8=184. Мы видим, что цифра 8 подходит. Можем ее записать в частном. 184-184=0. Деление выполнено без остатка.

Значение частного чисел 184 и 23 равно 8.

Рассмотрим более сложные случаи деления.

Найдем значение частного чисел 768 и 24.

Первое неполное делимое – 76 десятков. Значит, в частном будут 2 цифры.

Определим первую цифру частного. Разделим 76 на 24. Чтобы легче было подобрать цифру частного, разделим 76 не на 24, а на 20. То есть нужно 76 разделить на 10, будет 7 (остаток 6). И 7 разделим на 2, получится 3 (остаток 1). 3 – это пробная цифра частного. Сначала проверим, подойдет ли она. Умножим 24*3=72 . 76-72=4. Остаток меньше делителя. Значит, цифра 3 подошла и теперь мы ее можем записать на месте десятков частного. 72 пишем под первым неполным делимым, между ними ставим знак минус, под чертой записываем остаток.

Продолжим деление. Перепишем в строку с остатком цифру 8, следующую за первым неполным делимым. Получим следующее неполное делимое – 48 единиц. Разделим 48 на 24. Чтобы было легче подобрать цифру частного, разделим 48 не на 24, а на 20. То есть разделим 48 на 10, будет 4 (остаток 8). И 4 разделим на 2, будет 2. Это пробная цифра частного. Мы должны сначала проверить, подойдет ли она. Умножим 24*2=48. Мы видим, что цифра 2 подошла и, значит, можем ее записать на месте единиц частного. 48-48=0, деление выполнено без остатка.

Значение частного чисел 768 и 24 равно 32.

Найдем значение частного чисел 15344 и 56.

Первое неполное делимое – 153 сотни, значит, в частном будут три цифры.

Определим первую цифру частного. Разделим 153 на 56. Чтобы легче было подобрать цифру частного, разделим 153 не на 56, а на 50. Для этого разделим 153 на 10, будет 15 (остаток 3). И 15 разделим на 5, будет 3. 3 – это пробная цифра частного. Помните: ее нельзя сразу записывать в частном, а нужно сначала проверить, подойдет ли она. Умножим 56*3=168. 168 больше, чем 153. Значит, в частном будет меньше, чем 3. Проверим, подойдет ли цифра 2. Умножим 56*2=112. 153-112=41. Остаток меньше делителя, значит, цифра 2 подходит, ее можно записать на месте сотен в частном.

Образуем следующее неполное делимое. 153-112=41. Переписываем в ту же строку цифру 4, следующую за первым неполным делимым. Получаем второе неполное делимое 414 десятков. Разделим 414 на 56. Чтобы удобнее было подобрать цифру частного, разделим 414 не на 56, а на 50. 414:10=41(ост.4). 41:5=8(ост.1). Помните: 8 – это пробная цифра. Проверим ее. 56*8=448. 448 больше, чем 414, значит, в частном будет меньше, чем 8. Проверим, подойдет ли цифра 7. Умножим 56 на 7, получится 392. 414-392=22. Остаток меньше делителя. Значит, цифра подошла и в частном на месте десятков можем записать 7.

Пишем в строку с новым остатком 4 единицы. Значит следующее неполное делимое – 224 единицы. Продолжим деление. Разделим 224 на 56. Чтобы легче было подобрать цифру частного, разделим 224 на 50. То есть сначала на 10, будет 22 (остаток 4). И 22 разделим на 5, будет 4 (остаток 2). 4 – это пробная цифра, проверим ее, подойдет ли она. 56*4=224. И мы видим, что цифра подошла. Запишем 4 на месте единиц в частном. 224-224=0, деление выполнено без остатка.

Значение частного чисел 15344 и 56 равно 274.

Пример на деление с остатком

Чтобы провести аналогию, возьмем пример, похожий на пример выше, и отличающийся лишь последней цифрой

Найдем значение частного чисел 15345:56

Делим сначала точно так же, как в примере 15344:56, пока не дойдем до последнего неполного делимого 225. Разделим 225 на 56. Чтобы легче было подобрать цифру частного, разделим 225 на 50. То есть сначала на 10, будет 22 (остаток 5). И 22 разделим на 5, будет 4 (остаток 2). 4 – это пробная цифра, проверим ее, подойдет ли она. 56*4=224. И мы видим, что цифра подошла. Запишем 4 на месте единиц в частном. 225-224=1, деление выполнено с остатком.

Значение частного чисел 15345 и 56 равно 274 (остаток 1).

Деление с нулем в частном

Иногда в частном одним из чисел получается 0, и дети зачастую пропускают его, отсюда неправильное решение. Разберем, откуда может взяться 0 и как его не забыть.

Найдем значение частного чисел 2870:14

Первое неполное делимое — 28 сотен. Значит в частном будет 3 цифры. Ставим под уголок три точки. Это важный момент. Если ребенок потеряет ноль, останется лишняя точка, которая заставит задуматься, что где-то упущена цифра.

Определим первую цифру частного. Разделим 28 на 14. Подбором получается 2. Проверим, подойдет ли цифра 2. Умножим 14*2=28. Цифра 2 подходит, ее можно записать на месте сотен в частном. 28-28=0.

Получился нулевой остаток. Мы обозначили его розовым для наглядности, но записывать его не нужно. Переписываем в строку с остатком цифру 7 из делимого. Но 7 не делится на 14 с получением целого числа, поэтому записываем на месте десятков в частном 0.

Теперь переписываем в ту же строку последнюю цифру делимого (количество единиц).

70:14=5 Записываем вместо последней точки в частном цифру 5. 70-70=0. Остатка нет.

Значение частного чисел 2870 и 14 равно 205.

Деление нужно непременно проверить умножением.

Примеры на деление для самопроверки

Найдите первое неполное делимое и определите количество цифр в частном.

3432:66 2450:98 15145:65 18354:42 17323:17

Усвоили тему, а теперь потренируйтесь решить несколько примеров столбиком самостоятельно.

1428: 42 30296: 56 254415: 35 16514: 718

Удобно проводить особым методом, который получил название вычитание столбиком или вычитание в столбик . Этот способ вычитания оправдывает свое название, так как уменьшаемое, вычитаемое и разность записываются в столбик. Промежуточные вычисления также проводятся в столбиках, соответствующих разрядам чисел.

Удобство вычитания натуральных чисел столбиком заключается в простоте вычислений. Вычисления сводятся к использованию таблицы сложения и применению свойств вычитания.

Давайте разберемся, как выполняется вычитание столбиком. Процесс вычитания будем рассматривать вместе с решением примеров. Так будет понятнее.

Навигация по странице.

Что необходимо знать для вычитания столбиком?

Для вычитания натуральных чисел столбиком необходимо знать, во-первых, как выполняется вычитание с помощью таблицы сложения .

Наконец, не помешает повторить определение разряда натуральных чисел .

Вычитание столбиком на примерах.

Начнем с записи. Сначала записывается уменьшаемое. Под уменьшаемым располагается вычитаемое. Причем делается это так, что цифры оказываются одна под другой, начиная справа. Слева от записанных чисел ставится знак минус, а внизу проводится горизонтальная линия, под которой будет записан результат после проведения необходимых действий.

Приведем несколько примеров правильных записей при вычитании столбиком. Запишем в столбик разность 56−9 , разность 3 004−1 670 , а так же 203 604 500−56 777 .

Итак, с записью разобрались.

Переходим к описанию процесса вычитания столбиком. Его суть заключается в последовательном вычитании значений соответствующих разрядов. Сначала вычитаются значения разряда единиц, далее – значения разряда десятков, далее – значения разряда сотен и т.д. Результаты записываются под горизонтальной линией на соответствующих местах. Число, которое образуется под линией после завершения процесса, является искомым результатом вычитания двух исходных натуральных чисел.

Представим схему, иллюстрирующую процесс вычитания столбиком натуральных чисел.

Приведенная схема дает общую картину вычитания натуральных чисел столбиком, однако она не отражает всех тонкостей. С этими тонкостями разберемся при решении примеров. Начнем с самых простых случаев, а дальше будем постепенно продвигаться к более сложным случаям, пока не разберемся со всеми нюансами, которые могут встретиться при вычитании столбиком.

Пример.

Для начала вычтем столбиком из числа 74 805 число 24 003 .

Решение.

Запишем эти числа так, как этого требует метод вычитания столбиком:

Начинаем с вычитания значений разрядов единиц, то есть, вычитаем из числа 5 число 3 . Из таблицы сложения имеем 5−3=2 . Записываем полученные результат под горизонтальную черту в этом же столбике, в котором находятся числа 5 и 3 :

Теперь вычитаем значения разряда десятков (в нашем примере они равны нулю). Имеем 0−0=0 (это свойство вычитания мы упоминали в предыдущем пункте). Записываем полученный нуль под линию в том же столбике:

Идем дальше. Вычитаем значения разряда сотен: 8−0=8 (по свойству вычитания, озвученному в предыдущем пункте). Теперь наша запись примет следующий вид:

Переходим к вычитанию значений разряда тысяч: 4−4=0 (это свойств вычитания равных натуральных чисел). Имеем:

Осталось вычесть значения разряда десятков тысяч: 7−2=5 . Записываем полученное число под черту на нужное место:

На этом вычитание столбиком завершено. Число 50 802 , которое получилось внизу, является результатом вычитания исходных натуральных чисел 74 805 и 24 003 .

Рассмотрим следующий пример.

Пример.

Отнимем столбиком от числа 5 777 число 5 751 .

Решение.

Делаем все так же, как в предыдущем примере – вычитаем значения соответствующих разрядов. После завершения всех шагов запись примет следующий вид:

Под чертой получили число, в записи которого слева находятся цифры 0 . Если эти цифры 0 отбросить, то получим результат вычитания исходных натуральных чисел. В нашем случае отбрасываем две цифры 0 , получившиеся слева. Имеем: разность 5 777−5 751 равна 26 .

До этого момента мы вычитали натуральные числа, записи которых состоят из одинакового количества знаков. Сейчас на примере разберемся, как вычитаются столбиком натуральные числа, когда в записи уменьшаемого больше знаков, чем в записи вычитаемого.

Пример.

Вычтем из числа 502 864 число 2 330 .

Решение.

Записываем уменьшаемое и вычитаемое в столбик:

По очереди вычитаем значения разряда единиц: 4−0=4 ; далее – десятков: 6−3=3 ; далее – сотен: 8−3=5 ; далее – тысяч: 2−2=0 . Получаем:

Теперь, чтобы завершить вычитание столбиком, нам еще нужно вычесть значения разряда десятков тысяч, а дальше – значения разряда сотен тысяч. Но из значений этих разрядов (в нашем примере из чисел 0 и 5 ) нам вычитать нечего (так как вычитаемое число 2 330 не имеет цифр в этих разрядах). Как же быть? Очень просто – значения этих разрядов просто переписываются под горизонтальную линию:

На этом вычитание столбиком натуральных чисел 502 864 и 2 330 завершено. Разность равна 500 534 .

Осталось рассмотреть случаи, когда на некотором шаге вычитания столбиком значение разряда уменьшаемого числа меньше, чем значение соответствующего разряда вычитаемого. В этих случаях приходится «занимать» из старших разрядов. Давайте разберемся с этим на примерах.

Пример.

Вычтем столбиком из числа 534 число 71 .

Решение.

На первом шаге вычитаем из 4 число 1 , получаем 3 . Имеем:

На следующем шаге нам нужно вычитать значения разряда десятков, то есть, из числа 3 нужно вычесть число 7 . Так как 3, то мы не можем выполнить вычитание этих натуральных чисел (вычитание натуральных чисел определяется лишь когда вычитаемое не больше, чем уменьшаемое). Что же делать? В этом случае мы берем 1 единицу из старшего разряда и «размениваем» ее. В нашем примере «размениваем» 1 сотню на 10 десятков. Чтобы наглядно отразить наши действия, поставим жирную точку над числом в разряде сотен, а над числом в разряде десятков запишем число 10 , используя другой цвет. Запись примет следующий вид:

Прибавляем полученные после «размена» 10 десятков к 3 имеющимся десяткам: 3+10=13 , и из этого числа вычитаем 7 . Имеем 13−7=6 . Это число 6 записываем под горизонтальной чертой на свое место:

Переходим к вычитанию значений разряда сотен. Здесь мы видим над числом 5 точку, которая означает, что из этого числа мы брали единицу «на размен». То есть, сейчас мы имеем не 5 , а 5−1=4 . От числа 4 больше ничего отнимать не нужно (так как исходное вычитаемое число 71 не содержит цифр в разряде сотен). Таким образом, под горизонтальную черту записываем число 4 :

Итак, разность 534−71 равна 463 .

Иногда при вычитании столбиком «разменивать» единицы из старших разрядов приходится несколько раз. В подтверждение этих слов разберем решение следующего примера.

Пример.

Отнимем от натурального числа 1 632 число 947 столбиком.

Решение.

На первом же шаге нам нужно вычесть из числа 2 число 7 . Так как 2,то сразу приходится «разменивать» 1 десяток на 10 единиц. После этого из суммы 10+2 вычитаем число 7 , получаем (10+2)−7=12−7=5 :

На следующем шаге нам нужно вычесть значения разряда десятков. Мы видим, что над числом 3 стоит точка, то есть, мы имеем не 3 , а 3−1=2 . И от этого числа 2 нам нужно отнять число 4 . Так как 2, то опять приходится прибегать к «размену». Но сейчас уже размениваем 1 сотню на 10 десятков. При этом имеем (10+2)−4=12−4=8 :

Теперь вычитаем значения разряда сотен. Из числа 6 была занята единица на предыдущем шаге, поэтому имеем 6−1=5 . От этого числа нам нужно отнять число 9 . Так как 5, то нам нужно «разменять» 1 тысячу на 10 сотен. Получаем (10+5)−9=15−9=6 :

Остался последний шаг. Из единицы в разряде тысяч мы занимали на предыдущем шаге, поэтому имеем 1−1=0 . От полученного числа нам ничего больше отнимать не нужно. Это число и записываем под горизонтальную черту:

Калькулятор в столбик для Андроид устройств станет замечательным помощником для современных школьников. Программа не только дает правильный ответ на математическое действие, но и наглядно демонстрирует его пошаговое решение. Если же вам нужны более сложные калькуляторы – можете посмотреть или же продвинутый инженерный калькулятор.

Особенности

Главной особенностью программы является уникальность расчета математических операций. Отображение процесса вычислений столбиком дает возможность школьникам более подробно с ним ознакомиться, понять алгоритм решения, а не просто получить готовый результат и переписать его в тетрадь. Эта особенность имеет огромное преимущество перед другими калькуляторами, т.к. достаточно часто в школе учителя требуют расписать промежуточные вычисления, чтобы удостовериться, что школьник производит их в уме и действительно понимает алгоритм решения задач. Кстати, у нас есть еще одна программа похожего рода – .

Чтобы начать пользоваться программой, необходимо скачать калькулятор в столбик на Андроид. Сделать это можно на нашем сайте абсолютно бесплатно без дополнительных регистраций и смс. После установки откроется главная страница в виде тетрадного листа в клетку, на котором, собственно, и будут отображаться результаты вычислений и их подробное решение. Внизу располагается панель с кнопками:

  1. Цифры.
  2. Знаки арифметических действий.
  3. Удаление раннее введенных символов.

Ввод осуществляется по тому же принципу, что и на . Все отличие состоит только в интерфейсе приложения – все математические вычисления и их результат отображаются в виртуальной ученической тетради.

Приложение позволяет быстро и правильно выполнить стандартные для школьника математические вычисления столбиком:

  • умножение;
  • деление;
  • сложение;
  • вычитание.

Приятным дополнением в приложении является функция ежедневного напоминания о домашнем задании по математике. Хотите – делайте домашки. Для ее включения следует зайти в настройки (нажать кнопку в виде шестеренки) и установить галочку о напоминании.

Достоинства и недостатки

  1. Помогает школьнику не просто быстро получить правильный результат математических вычислений, но и понять сам принцип расчета.
  2. Очень простой, интуитивно понятный интерфейс для каждого пользователя.
  3. Установить приложение можно даже на самое бюджетное Андроид устройство с операционной системой 2.2 и более поздней версией.
  4. Калькулятор сохраняет историю проведенных математических вычислений, которую можно в любой момент очистить.

Калькулятор ограничен в математических операциях, поэтому применить его для сложных расчетов, с какими мог бы справиться инженерный калькулятор, не получится. Однако учитывая назначение самого приложения – наглядно продемонстрировать учащимся младшей школы принцип расчета в столбик, считать это недостатком не стоит.

Приложение также станет отличным помощником не только для школьников, но и для родителей, которые желают заинтересовать своего ребенка математикой и научить его правильно и последовательно производить вычисления. Если Вы уже пользовались приложением Калькулятор в столбик, оставьте свои впечатления ниже в комментариях.

Научить ребенка делению столбиком просто. Необходимо объяснить алгоритм этого действия и закрепить пройденный материал.

  • Согласно школьной программе, деление столбиком детям начинают объяснять уже в третьем классе. Ученики, которые схватывают все «на лету», быстро понимают эту тему
  • Но, если ребенок заболел и пропустил уроки математики, или он не понял тему, тогда родители должны самостоятельно малышу объяснить материал. Нужно максимально доступно донести до него информацию
  • Мамы и папы во время учебного процесса ребенка должны быть терпеливыми, проявляя такт по отношению к своему чаду. Ни в коем случае нельзя кричать на ребенка, если у него что-то не получается, ведь так можно отбить у него всю охоту к занятиям



Важно: Чтобы ребенок понял деление чисел, он должен досконально знать таблицу умножения. Если малыш плохо знает умножение, он не поймет деление.

Во время домашних дополнительных занятий можно пользоваться шпаргалками, но ребенок должен выучить таблицу умножения, прежде чем, приступать к теме «Деление».

Итак, как объяснить ребенку деление столбиком :

  • Постарайтесь сначала объяснить на маленьких цифрах. Возьмите счетные палочки, например, 8 штук
  • Спросите у ребенка, сколько пар в этом ряду палочек? Правильно — 4. Значит, если разделить 8 на 2, получится 4, а при делении 8 на 4 получится 2
  • Пусть ребенок сам разделит другое число, например, более сложное: 24:4
  • Когда малыш освоил деление простых чисел, тогда можно переходить к делению трехзначных чисел на однозначные



Деление всегда дается детям немного сложнее, чем умножение. Но усердные дополнительные занятия дома помогут малышу понять алгоритм этого действия и не отставать от сверстников в школе.

Начинайте с простого — деление на однозначное число:

Важно: Просчитайте в уме, чтобы деление получилось без остатка, иначе ребенок может запутаться.

Например, 256 разделить на 4:

  • Начертите на листе бумаги вертикальную линию и разделите ее с правой части пополам. Слева напишите первую цифру, а справа над чертой вторую
  • Спросите у малыша, сколько четверок помещается в двойке — нисколько
  • Тогда берем 25. Для наглядности отделите это число сверху уголком. Опять спросите у ребенка, сколько помещается четверок в двадцати пяти? Правильно — шесть. Пишем цифру «6» в правом нижнем углу под линией. Ребенок должен использовать таблицу умножения для правильного ответа
  • Запишите под 25 цифру 24, и подчеркните, чтобы записать ответ — 1
  • Опять спрашивайте: в единице сколько помещается четверок — нисколько. Тогда сносим к единице цифру «6»
  • Получилось 16 — сколько четверок помещается в этом числе? Правильно — 4. Записываем «4» рядом с «6» в ответе
  • Под 16 записываем 16, подчеркиваем и получается «0», значит мы разделили правильно и ответ получился «64»

Письменное деление на двузначное число



Когда ребенок освоил деление на однозначное число, можно двигаться дальше. Письменное деление на двузначное число чуть сложнее, но если малыш поймет, как производится это действие, тогда ему не составит труда решать такие примеры.

Важно: Снова начинайте объяснять с простых действий. Ребенок научится правильно подбирать цифры и ему будет легко делить сложные числа.

Выполните вместе такое простое действие: 184:23 — как нужно объяснять:

  • Разделим сначала 184 на 20, получается примерно 8. Но мы не пишем цифру 8 в ответ, так как это пробная цифра
  • Проверяем, подходит 8 или нет. Умножаем 8 на 23, получается 184 — это именно то число, которое у нас стоит в делителе. Ответ будет 8

Важно: Чтобы ребенок понял, попробуйте вместо восьмерки взять 9, пусть он умножит 9 на 23, получается 207 — это больше, чем у нас в делителе. Цифра 9 нам не подходит.

Так постепенно малыш поймет деление, и ему будет легко делить более сложные числа:

  • Разделим 768 на 24. Определите первую цифру частного — делим 76 не на 24, а на 20, получается 3. Записываем 3 в ответ под чертой справа
  • Под 76 записываем 72 и проводим линию, записываем разность — получилось 4. Эта цифра делится на 24? Нет — сносим 8, получается 48
  • Цифра 48 делится на 24? Правильно — да. Получается 2, записываем эту цифру в ответ
  • Получилось 32. Теперь можно проверить — правильно ли мы выполнили действие деления. Сделайте умножение в столбик: 24х32, получается 768, значит все правильно



Если ребенок научился выполнять деление на двузначное число, тогда необходимо перейти к следующей теме. Алгоритм деления на трехзначное число такой же, как и алгоритм деления на двузначное число.

Например:

  • Разделим 146064 на 716. Берем сначала 146 — спросите у ребенка делится это число на 716 или нет. Правильно — нет, тогда берем 1460
  • Сколько раз число 716 поместится в числе 1460? Правильно — 2, значит пишем эту цифру в ответе
  • Умножаем 2 на 716, получается 1432. Записываем эту цифру под 1460. Получается разность 28, записываем под чертой
  • Сносим 6. Спросите у ребенка — 286 делится на 716? Правильно — нет, поэтому пишем 0 в ответе рядом с 2. Сносим еще цифру 4
  • Делим 2864 на 716. Берем по 3 — мало, по 5 — много, значит получается 4. Умножаем 4 на 716, получается 2864
  • Запишите 2864 под 2864, получается в разности 0. Ответ 204

Важно: Для проверки правильности выполнения деления, умножьте вместе с ребенком в столбик — 204х716=146064. Деление выполнено правильно.



Пришло время ребенку объяснить, что деление может быть не только нацело, но и с остатком. Остаток всегда меньше делителя или равен ему.

Деление с остатком следует объяснять на простом примере: 35:8=4 (остаток 3):

  • Сколько восьмерок помещается в 35? Правильно — 4. Остается 3
  • Делится эта цифра на 8? Правильно — нет. Получается, остаток 3

После этого ребенок должен узнать, что можно продолжать деление, дописывая 0 к цифре 3:

  • В ответе стоит цифра 4. После нее пишем запятую, так как добавление нуля говорит о том, что число будет с дробью
  • Получилось 30. Делим 30 на 8, получается 3. Записываем в ответ, а под 30 пишем 24, подчеркиваем и пишем 6
  • Сносим к цифре 6 цифру 0. Делим 60 на 8. Берем по 7, получается 56. Пишем под 60 и записываем разность 4
  • К цифре 4 дописываем 0 и делим на 8, получается 5 — записываем в ответ
  • Вычитаем 40 из 40, получается 0. Итак, ответ: 35:8=4,375



Совет: Если ребенок что-то не понял — не злитесь. Пусть пройдет пару дней и снова постарайтесь объяснить материал.

Уроки математики в школе также будут закреплять знания. Пройдет время и малыш будет быстро и легко решать любые примеры на деление.

Алгоритм деления чисел заключается в следующем:

  • Сделать прикидку числа, которое будет стоять в ответе
  • Найти первое неполное делимое
  • Определить число цифр в частном
  • Найти цифры в каждом разряде частного
  • Найти остаток (если он есть)

По такому алгоритму выполняется деление как на однозначные числа, так и на любое многозначное число (двузначное, трехзначное, четырехзначное и так далее).



Занимаясь с ребенком, чаще ему задавайте примеры на выполнение прикидки. Он должен быстро в уме подсчитать ответ. Например:

  • 1428:42
  • 2924:68
  • 30296:56
  • 136576:64
  • 16514:718

Для закрепления результата можно использовать такие игры на деление:

  • «Головоломка». Напишите на листе бумаги пять примеров. Только один из них должен быть с правильным ответом.

Условие для ребенка: Среди нескольких примеров, только один решен правильно. Найди его за минуту.

Видео: Игра арифметика для детей сложение вычитание деление умножение

Видео: Развивающий мультфильм Математика Изучение наизусть таблицы умножения и деления на 2

Как вычитать столбиком

Вычитание многозначных чисел обычно выполняют столбиком, записывая числа друг под другом (уменьшаемое сверху, вычитаемое снизу) так, чтобы цифры одинаковых разрядов стояли друг под другом (единицы под единицами, десятки под десятками и т. д.). Слева между числами ставится знак действия. Под вычитаемым проводят черту. Вычисление начинают с разряда единиц: из единиц вычитают единицы, затем из десятков — десятки и т. д. Результат вычитания записывают под чертой:

Рассмотрим пример, когда в каком-либо разряде цифра уменьшаемого меньше цифры вычитаемого:

От 2 мы не можем отнять 9, что нам делать в этом случае? В разряде единиц у нас нехватка, но в разряде десятков у уменьшаемого аж 7 десятков, поэтому мы можем один из этих десятков перекинуть в разряд единиц:

В разряде единиц у нас было 2, мы перекинули десяток, стало 12 единиц. Теперь мы легко можем от 12 отнять 9. Записываем под чертой в разряде единиц 3. В разряде десятков у нас было 7 единиц, одну из них мы перекинули в простые единицы, осталось 6 десятков. Записываем под чертой в разряде десятков 6. В результате мы получили число 63:

Вычитание столбиком обычно не записывают так подробно, вместо этого, над цифрой разряда, у которого будет занята единица, ставят точку, чтобы не запоминать, у какого разряда надо будет дополнительно вычесть единицу:

При этом говорят так: из 2 вычесть 9 нельзя, занимаем единицу, из 12 вычитаем 9 — получим 3, пишем 3, в разряде десятков у нас было 7 единиц, мы одну перекинули, осталось 6, пишем 6 .

Теперь рассмотрим вычитание столбиком из чисел, содержащих нули:

Начинаем вычитать. От 7 отнимаем 3, пишем 4. От нуля мы не можем отнять 5, поэтому мы вынуждены занять единицу в старшем разряде, но в старшем разряде у нас тоже 0, поэтому и для этого разряда мы вынуждены занять в более старшем разряде. Занимаем единицу из разряда тысяч, получаем 10 сотен:

Одну из единиц разряда сотен мы занимаем в младший разряд, получаем 10 десятков. Из 10 вычитаем 5, пишем 5:

В разряде сотен у нас осталось 9 единиц поэтому, от 9 отнимаем 6, пишем 3. В разряде тысяч у нас была единица, но мы её потратили на младшие разряды, поэтому здесь остаётся нуль (его записывать не надо). В результате мы получили число 354:

Такая подробная запись решения была приведена, чтобы было проще понять, как выполняется вычитание столбиком из чисел содержащих нули. Как уже упоминалось, на практике решение обычно записывается так:

А все упомянутые действия выполняются в уме. Чтобы было легче выполнять вычитание, запомните простое правило:

Если при вычитании столбиком над нулём стоит точка, нуль превращается в 9.

Калькулятор вычитания столбиком

Данный калькулятор поможет вам выполнить вычитание чисел столбиком. Просто введите уменьшаемое и вычитаемое и нажмите кнопку Вычислить.

Презентация по математике на тему Письменные приемы сложения и вычитания многозначных чисел 4 класс доклад, проект

  • Главная
  • Разное
  • Образование
  • Спорт
  • Естествознание
  • Природоведение
  • Религиоведение
  • Французский язык
  • Черчение
  • Английский язык
  • Астрономия
  • Алгебра
  • Биология
  • География
  • Геометрия
  • Детские презентации
  • Информатика
  • История
  • Литература
  • Математика
  • Музыка
  • МХК
  • Немецкий язык
  • ОБЖ
  • Обществознание
  • Окружающий мир
  • Педагогика
  • Русский язык
  • Технология
  • Физика
  • Философия
  • Химия
  • Шаблоны, фоны, картинки для презентаций
  • Экология
  • Экономика

Презентация на тему Презентация по математике на тему Письменные приемы сложения и вычитания многозначных чисел 4 класс, предмет презентации: Математика.  Этот материал в формате pptx (PowerPoint) содержит 18 слайдов, для просмотра воспользуйтесь проигрывателем. Презентацию на заданную тему можно скачать внизу страницы, поделившись ссылкой в социальных сетях! Презентации взяты из открытого доступа или загружены их авторами, администрация сайта не отвечает за достоверность информации в них, все права принадлежат авторам презентаций и могут быть удалены по их требованию.

Слайд 1
Текст слайда:

Прозвенел уже звонок,
начинается урок.
По местам скорей садитесь,
Новых знаний наберитесь.


Слайд 2
Текст слайда:

26 ноября
Классная работа


Слайд 3
Текст слайда:

345 + 345
3456 + 3456
34567 + 34567

Чем отличаются примеры?


Слайд 4
Текст слайда:

Открытый урок

Тема:»Сложение и вычитание многозначных чисел».

— познакомиться с алгоритмами сложения и вычитания многозначных чисел;
— повторить устную и письменную нумерацию и сравнение многозначных чисел,
— тренировать вычислительные навыки

 


Слайд 5
Текст слайда:

Прочитайте числа

5 901 006
312 096 312
45 003 420

Назовите единицы ΙΙΙ, ΙΙ, Ι классов.

Какое из чисел самое большое?


Слайд 6
Текст слайда:

438

257

695


Слайд 7
Текст слайда:

941

386

555


Слайд 8
Текст слайда:

10386

86310

96696


Слайд 9
Текст слайда:

86310

10368

75942


Слайд 10

Слайд 11

Слайд 12
Текст слайда:

Реши примеры столбиком .

737246 + 54337= 392476 + 1346=
358219 + 12275= 516739 + 175152=


Слайд 13
Текст слайда:

Реши примеры столбиком .
737246 + 54337= 791583
392476 + 1346=393822
358219 + 12275=370494
516739 + 175152=691891


Слайд 14
Текст слайда:

В магазин привезли шампиньоны. В течение суток продали 176 кг грибов. Грибов осталось на 145 кг меньше, чем продали. Сколько в магазин привезли грибов?


Слайд 15
Текст слайда:

Привезли — ? кг
Продали – 176кг.
Осталось — ? кг., на 145 кг.˂

— Сможем мы сразу ответить на вопрос задачи?
— Что нам надо узнать сначала?
— Запишем вопрос и решение.
— А теперь мы сможем ответить на вопрос задачи?
— Запишем вопрос и решение.


Слайд 16
Текст слайда:

Решение.
1)176 – 145=31(кг) осталось
2)176+31 =207 (кг) привезли.
Ответ: 207 килограммов


Слайд 17

Слайд 18
Текст слайда:

Домашнее задание

Учебник: стр 60
№ 262, №264


Скачать презентацию

Что такое shareslide.ru?

Это сайт презентаций, где можно хранить и обмениваться своими презентациями, докладами, проектами, шаблонами в формате PowerPoint с другими пользователями. Мы помогаем школьникам, студентам, учителям, преподавателям хранить и обмениваться учебными материалами.


Для правообладателей

Обратная связь

Email: Нажмите что бы посмотреть 

Примеры на сложение и вычитание трёхзначных чисел

Примеры на сложение и вычитание трёхзначных чисел

ПримерОнлайн.ru

Генератор примеров по математике 1-3 класс

Онлайн примеры на сложение трёхзначных чисел позволяют вывести большое количество неповторяющихся примеров с трёхзначными числами на сложение и вычитание.

Сочетание примеров позволяет выработать навыки устного счёта, и закрепить их большим количеством решённых примеров.

Примеры можно разделить по степени сложности: лёгкие – это примеры без перехода через десяток, сложные – с обязательным переходом, обычные – слагаемые выбираются случайным образом.

 



Настройка генератора примеров



Файл для печати

В файле 1234567891012152025303540455060708090100 стр.

Скачать: файл заданий, файл ответов

Свой формат страницы

Выводить 153050100200300500 примеров

Распечатать страницу

Интерактивные примеры

Выводить 153050100200300500 примеров

Показать страницу

Образец примеров



846 — 724

251 + 280

851 — 408

744 — 514

266 — 233

881 — 648

748 + 179

217 + 133

935 — 200

160 + 592

164 + 214

734 + 240

298 + 300

717 — 200

520 + 474

356 + 600

551 — 177

550 — 371

402 — 109

968 — 721

253 + 653

382 + 476

541 — 518

723 — 619

381 — 237

786 — 199

125 + 183

563 + 338

507 — 177

132 + 748

406 + 251

958 — 954

622 + 367

654 — 242

364 + 622

113 + 353

949 — 155

808 + 126

919 — 630

108 + 890

677 — 339

338 + 199

743 — 739

410 + 299

773 — 201

582 + 351

461 + 258

668 + 169

146 + 265

336 + 587

419 + 527

136 + 641

185 + 449

100 + 755

788 — 575

508 — 173

336 — 144

471 + 350

202 + 778

624 — 308

186 + 389

239 + 159

930 — 915

422 + 152

342 — 300

915 — 336

931 — 887

819 — 489

130 + 218

407 + 431

412 + 264

921 — 659

819 — 688

833 — 105

628 — 140

514 + 440

416 + 298

151 + 369

215 + 331

634 + 187

137 + 668

137 + 437

289 + 587

842 — 206

284 — 273

457 + 101

911 — 464

468 + 227

535 + 338

150 + 781

345 + 321

549 — 501

954 — 107

568 + 176

238 + 669

175 + 779

285 + 391

334 + 347

112 + 431

806 — 498

656 + 114

371 + 161

198 + 207

839 — 677

861 — 363

404 + 358

412 + 201

188 + 188

653 — 414

417 + 363

380 + 199

784 — 649

472 — 409

241 + 115

157 + 305

612 + 275

164 + 835

466 — 386

769 + 203

743 — 708

801 + 105

354 — 224

186 + 373

374 + 321

842 + 157

658 + 207

395 — 114

495 + 435

183 + 513

784 — 760

409 — 183

450 + 417

939 — 880

393 + 281

758 — 251

391 — 106

876 — 189

590 — 377

334 — 210

786 — 111

256 + 688

931 — 861

600 — 473

354 + 352

242 + 324

167 + 812

868 — 369

518 — 273

764 + 147

343 — 343

581 + 352

253 — 134

580 + 335

766 — 428

758 — 703

849 — 347

104 + 760

469 + 322

600 + 104

204 + 536

737 + 162

201 + 201

436 — 112

134 + 435

980 — 482

940 — 515

360 + 185

308 + 619

900 — 666

443 — 201

124 + 299

547 — 461

214 + 673

754 — 223

480 — 290

495 + 448

569 — 131

263 + 532

718 — 165

176 + 733

203 + 475

859 + 135

327 + 324

453 + 303

498 + 447

158 + 297

397 — 356

152 + 742

279 + 637

306 + 218

935 — 825

365 + 442

226 + 571

765 + 148

607 — 195

669 — 410

225 — 224

938 — 554

674 — 120

871 — 690

617 — 452

930 — 234

520 — 149

461 + 129

221 + 177

122 + 796

798 — 111

511 — 239

108 + 182

825 — 615

206 + 456

987 — 740

222 + 386

254 + 688

337 — 199

238 + 440

450 + 171

448 + 198

604 — 412

877 — 218

196 + 215

906 — 530

767 — 360

222 + 697

642 + 331

350 — 199

363 + 215

197 + 119

626 — 541

403 + 362

368 + 495

105 + 581

236 — 207

779 — 109

822 — 182

118 + 565

884 — 467

533 — 364

697 — 137

730 — 358

947 — 875

496 — 398

316 + 180

961 — 114

458 + 524

887 — 816

735 — 130

126 + 195

240 + 516

563 — 416

562 — 385

211 + 306

656 + 102

911 — 471

725 — 706

905 — 730

915 — 379

389 + 506

962 — 645

904 — 479

119 + 133

722 — 565

123 + 814

351 — 149

829 — 291

397 + 601

332 + 191

317 + 260

717 — 650

767 — 528

861 — 726

571 — 113

444 + 274

459 — 294

457 + 506

151 + 499

564 — 245

567 — 132

554 — 479

694 — 465

551 — 288

421 — 126

180 + 381

692 — 176

110 + 492

304 + 439

413 — 230

341 + 622

394 + 280

892 — 105

306 + 428

420 + 538

295 + 671

770 — 463

157 + 495

366 + 261

590 — 209

162 + 359

579 + 307

320 + 262

295 — 163

600 — 141

959 — 523

985 — 838

132 + 195

182 + 500

765 — 546

433 + 118

340 — 272

449 — 201

435 + 438

493 — 305

156 + 192

345 — 149

446 + 234

718 — 433

226 — 218

671 — 329

504 + 455

885 — 796

409 — 350

460 + 423

897 — 651

166 + 316

595 — 576

128 + 846

813 — 795

256 + 659

703 + 158

675 + 179

373 + 395

336 + 161

265 + 316

630 — 422

833 — 277

355 + 309

100 + 683

565 + 304

879 — 222

185 — 104

724 — 642

336 — 117

605 — 174

845 + 149

590 — 453

881 — 135

434 + 516

303 + 364

614 — 137

922 — 544

486 — 202

546 — 129

377 — 314

107 + 504

962 — 860

899 — 829

943 — 311

474 — 258

898 — 252

387 + 511

455 — 441

581 + 406

190 + 445

897 — 542

664 + 160

647 — 490

485 — 135

225 + 453

928 — 527

331 + 236

510 + 353

186 + 137

955 — 746

241 + 210

667 + 103

102 + 837

182 + 579

519 — 484

856 — 737

522 + 293

384 + 231

258 — 218

904 — 445

436 + 241

430 — 106

583 + 386

899 — 156

103 + 176

164 — 108

527 + 211

125 + 520

557 + 212

549 + 122

395 + 449

979 — 804

769 — 559

593 — 318

716 — 369

239 + 685

189 + 675

172 + 124

267 + 308

963 — 427

142 + 454

926 — 919

425 + 208

285 + 383

447 + 182

999 — 903

167 + 258

417 + 245

885 — 292

447 + 171

407 — 282

824 — 136

723 — 226

579 — 277

454 + 313

451 + 351

957 — 775

241 — 233

656 + 183

279 + 543

472 + 181

718 + 281

104 + 317

612 + 176

830 — 317

301 + 634

264 + 558

138 + 224

157 + 177

995 — 297

687 — 257

136 + 160

618 — 545

253 + 293

336 + 348

402 + 410

100 + 633

289 + 682

736 — 529

167 + 275

271 + 569

464 — 464

396 — 130

653 — 246

980 — 334

642 — 238

130 + 556

258 — 189

653 — 355

248 + 341

788 — 524

925 — 911

212 — 212

882 — 150

834 — 499

416 + 108

912 — 314

277 — 129

748 — 496

253 — 162

571 + 103

353 + 522

809 — 174

813 — 243

693 — 218

530 — 325

746 — 364

255 + 621

287 — 181

281 + 350

615 — 255

985 — 231

446 + 244

405 + 130

840 — 613

172 + 610

348 — 274

158 + 238

737 — 317

661 + 238

663 — 556

358 + 414

372 + 300

353 + 599

464 + 314

566 — 555

320 + 374

389 — 247

863 — 572

371 — 312

396 + 397

191 + 439

339 + 510

160 + 671

148 + 415

 

Что такое вычитание столбцов? Объяснение для начальной школы

Здесь мы объясняем, что такое вычитание по столбцам, рассказываем, как его используют и изучают в начальных школах, и приводим несколько примеров вопросов, связанных с этим методом!

Вычитание в столбик — это лишь часть пути ребенка к вычитанию. Это также часть пути, на котором учителя не должны спешить с подведением учащихся к этому этапу. Нашим младшим учащимся было бы намного лучше, если бы они познакомились и имели достаточно времени для практики умственных методов вычитания, которые были тщательно продуманы и упорядочены. Только в 3-м классе необходимо вводить «формальный» метод вычитания столбцов.

Что такое вычитание столбцов?

Вычитание по столбцам – это то, что в учебной программе называется «формальным» письменным методом решения вопросов и задач, связанных с вычитанием. Метод вычитания по столбцам впервые вводится в 3-м классе, но будет продолжать использоваться учащимися до 6-го класса и далее.

Метод вычитания в столбцах основан на способности учащихся правильно распределять вычисления, например 653–321, в столбцы, чтобы можно было выполнить операцию вычитания.

Пример этого можно найти в приложении 1 к учебной программе по математике:

Следует иметь в виду, что до того, как учащиеся достигнут этой ступени, им необходимо овладеть многими другими математическими навыками.

Что метод расширенного столбца?

Прежде чем перейти непосредственно к описанному выше методу столбцов, хорошим промежуточным способом подготовить учеников к нему является использование метода расширенных столбцов.

«Расширенный» относится к записи каждой цифры в соответствующее значение.

Например, запись 189 в расширенной форме будет 100, 80 и 9. В контексте вычитания столбцов это будет выглядеть так.

789 – 458:

700 80 9
– 400 50 8
= 300 30 1

= 331

Однако в национальной учебной программе нет требования, чтобы учителя преподавали по расширенному методу.

Присоединяйтесь к математическому центру Third Space Learning

Чтобы просмотреть всю нашу коллекцию бесплатных и платных математических ресурсов для учителей и родителей, зарегистрируйтесь, чтобы присоединиться к математическому центру Third Space Learning. Это быстро, легко и бесплатно! (Пожалуйста, используйте Google Chrome для доступа к Maths Hub)

Когда дети узнают о вычитании в столбик в школе?

Дети впервые познакомятся с вычитанием в столбцах в 3-м классе. Ожидается, что они будут:

  • складывать и вычитать числа, содержащие до 3 цифр, используя формальные письменные методы сложения и вычитания в столбцах

В 4-м классе это прогрессирует к следующему:

  • складывать и вычитать числа, содержащие до 4 цифр, используя формальные письменные методы столбцового сложения и вычитания, где это применимо

В 5-м классе учащиеся должны:

  • складывать и вычитать целые числа, состоящие более чем из 4 цифр, в том числе с использованием формальных письменных методов (сложение и вычитание в столбцах)

Для 6-го класса, пока нет

  • Учащиеся практикуют сложение, вычитание, умножение и деление больших чисел, используя формальные письменные методы сложения и вычитания в столбцах, короткого и длинного умножения и короткое и длинное деление  

Как расположить и вычесть столбцов?

Когда вы переставляете письменный расчет вычитания в столбцы, важно помнить некоторые важные элементы, которые должны быть правильными.

В первую очередь важен порядок каждой цифры. Возьмем 789 – 458. Первое число называется уменьшаемым, а второе число – вычитаемым.

Важно, чтобы при написании метода столбца уменьшаемое было верхним числом, а вычитаемое — нижним.

Например, 789 – 458 дадут совсем другой ответ, чем 458 – 789.

Еще одна важная часть упорядочения – убедиться, что значения разрядов совпадают. Это особенно верно, когда вам нужно вычесть числа, которые имеют разное количество цифр. Неправильное выполнение этой части процесса обязательно приведет к тому, что процедура выдаст неверный ответ.

Если вычисление было 2,345 – 567, обязательно, чтобы они были расположены правильно, особенно чтобы 5 помещалось в столбец сотен, а не в столбец тысяч, как показано ниже.

2 345, а не как 2 345

– 567                – 567

Вот почему обязательным условием вычитания столбцов является четкое понимание разрядного значения. Вот почему расширенный метод, как упоминалось выше, может быть полезной стратегией для включения.

Перегруппировка

Одним из аспектов, который поначалу вызывает трудности у учащихся, является концепция перегруппировки. Это происходит, когда цифра в вычитаемом больше, чем цифра того же разряда в уменьшаемом. Например:

783  –

458

Поскольку 8 больше 3, необходимо использовать стратегию перегруппировки. Делать это с помощью счетчиков позиционных значений может быть полезно для математического представления того, что происходит.

Это показывает 783 и показывает, что 8 единиц нельзя убрать, так как их всего 3.

Теперь мы обменяли одну из десяток на 10 единиц. Это означает, что теперь у меня есть 13 единиц, и это позволяет вычесть 8.

Вы можете видеть, что число, представленное в настоящее время, теперь равно 775. Теперь я могу продолжить вычисления, вычитая 50 и вычитая 400.

С вычитанием 50.

С вычитанием 400.

Это представлено в столбце вычитания следующим образом.

Вычитание Двухзначные числа из Трехзначные числа

трехзначные числа.

Мы хотим, чтобы учащиеся использовали следующее:

345, а не как 345

  67                       67

Использование расширенного вычитания столбцов помогает учащимся избежать этой ошибки, поскольку значение места каждой цифры становится более четким. 345 – 67, написанное в расширенной форме, будет 300 40 5 – 60 7 =

300 40 5 –

        60 7

Предоставление учащимся выполнения некоторых расчетов с использованием этого метода перед переходом к нестандартному методу может смягчить эту проблему. Вы также можете предложить учащимся представить вычисление с помощью счетчиков разряда. Обратите внимание, что это отличается от использования манипуляции, чтобы помочь им решить фактический расчет.

345 минус 67 показано с использованием счетчиков мест.

Когда учащиеся могут удобно записать вычисление следующим образом:

345 –

67   

Тогда процесс решения задачи становится таким же, как решение вычисления с перегруппировкой. В зависимости от того, где находятся учащиеся, им могут понадобиться манипуляторы, чтобы помочь им решить эту проблему.

Слайд из онлайн-вмешательства Third Space Learning по математике, в котором счетчики разрядов используются для обучения 5-х классов вычитанию столбцов и перегруппировке.

Вычитание столбцов с десятичными дробями

Когда учащиеся улучшат свое понимание десятичных разрядов, они должны будут использовать свои знания об этом и о вычитании столбцов для решения задач, связанных с десятичными дробями. Обычно это делается в контексте измерений.

Важно, чтобы учащиеся понимали, что принципы разряда и то, как работает вычитание столбцов, не меняются, меняется только контекст (десятичные числа).

Как вычитание столбцов связано с реальной жизнью?

Вычитание столбцов в значительной степени относится к реальной жизни, поскольку изо дня в день мы постоянно используем вычитание и сложение для больших чисел при планировании бюджета, проверке нашего банковского баланса, счетов или платежных ведомостей.

Для школьников можно использовать примеры того, как получить карманные деньги и посмотреть, сколько осталось, тратя их на сладости или поход в кино. Вы также можете связать вычитание столбца, чтобы рассчитать расстояние, оставшееся до пункта назначения.

Как вычитание столбцов связано с другими областями математики?

Вычитание в столбик является фундаментальным навыком для учащихся и может использоваться почти во всех областях математики, особенно при вычислениях и решении задач с измерениями.

Какие еще методы вычитания используются в школах?

Как упоминалось во введении, вычитание по столбцам не должно быть основной стратегией, которой сразу же обучают студентов. Есть много других умственных стратегий, которым учащиеся должны научиться в первую очередь, и которые они используют на ранних этапах школьного обучения. Общие стратегии включают в себя:

  • Разбиение вычитаемого: Здесь вычитаемое разбивается на разные разрядные значения. Затем это используется учащимися для выполнения двух отдельных вычитаний.

Например:

78 – 36 =

78 – 30 и  6. 

6 можно вычесть из 8, чтобы получить 72. Затем 30 можно вычесть из 72, чтобы получить 42.

0 до десяти: . Это связано с разбиением, но число не разбивается на его расширенную форму, а разбивается таким образом, что оно переводит уменьшаемое до следующего кратного 10. Например, в 165 – 7 7 может можно разделить на 5 и 2. Из 165 можно вычесть 5, чтобы получить 160. Наконец, из 160 можно вычесть 2, чтобы получить 158. Числовые линии особенно полезны для первоначального изучения этой стратегии, но учащимся не следует полагаться на них. .
  • Та же разница: Эта стратегия особенно полезна при вычитании числа из уменьшаемого, которое требует большого количества перегруппировок. Например. 2000 – 256. Тот же принцип разности применим к вычитанию следующим образом:

    Представьте, что это 10 – 8. Разница равна 2. 2 также является разницей между 9 и 7, 8 и 6 и т. д. Во всех случаях уменьшаемое и вычитаемое уменьшается на 1, и это приводит к той же разнице.

    С 2000 – 256 мы можем применить тот же принцип и уменьшить оба числа на 1, зная, что разница останется прежней. Теперь расчет становится 1999 – 255 =. Теперь учащиеся могут вычитать и полагаться на факты вычитания, с которыми они гораздо лучше знакомы. 9 – 5 = 4, 90-50 = 40 и 900 минус 700 равно 200. 1000 останется прежним, поэтому окончательный ответ будет 1744. Это можно записать следующим образом:

    2000 – 256 = 1999 – 255 = 1744

Когда учащиеся изучают формальный метод вычитания, у них появляется привычка использовать этот метод для решения каждого вычитания, с которым они сталкиваются. Учителя, однако, должны поощрять использование формального письменного метода в качестве отступления или стратегии проверки и отдавать приоритет умственным стратегиям на ключевом этапе 29.0005

Примеры работы вычитания столбцов

1. 2874 – 123 =

Простой вопрос, на который, если вы не убедитесь, что вычитаемое и уменьшаемое правильно расположены, приведет к публикации относительных проблем. Это тот тип вопросов, когда мы должны поощрять студентов сначала попытаться ответить в уме.

2. 4,783 – 2,349

4,783 – 

2,349

Здесь студенты увидят, что им нужно обменять 1 десяток на 10 единиц. Это даст следующее:

Далее процедура может продолжаться как обычно, работая от самого низкого разряда до самого высокого.

После обмена дополнительных сложностей не возникает.

3. 2 354,43 – 1 789,52 = 

Начало этого процесса простое и должно быть знакомо читателям, читавшим блог.

Дополнительные трудности указаны в следующей колонке.

Здесь учащийся должен поменять местами столбец десятков и перенести 1 десяток в столбец единиц, зная, что из-за предыдущего шага столбец единиц имеет значение 3, а не 4.

В остальном проблема повторяется.

Задачи на вычитание столбцов и ответы

1. На банковском счете Гарри было 365 фунтов стерлингов. Его мама позволила ему потратить 249 фунтов стерлингов на новый велосипед. Сколько осталось на его счету?

Этот вопрос является типичным примером вычитания, который демонстрирует структуру выноса. Физическая величина отнимается от большей величины, и поэтому важно правильно организовать это.

2. Книга Билла состояла из 561 слова. Книга Теда состояла из 395 слов. На сколько больше слов прочитал Билл, чем Тед?

Этот вопрос является примером структуры вычитания разницы. Мы не отнимаем одну величину от другой, а скорее рассматриваем разницу между двумя величинами. Примеры вопросов на вычитание из столбцов0004

(ответ = 2 186)

3. 795.32 — 452,12 =

(ответ = 343.11)

4. У Гарри было 734 фунтов стерлингов в своей банковской счете. Его мама позволила ему потратить 375 фунтов стерлингов на новый велосипед. Сколько осталось на его счету?

(Ответ: 359 фунтов стерлингов)

5. Книга Билла состояла из 1453 слов. Книга Теда состояла из 954 слов. На сколько больше слов прочитал Билл, чем Тед?

(ответ = 499)

Как вы объясните вычитание по столбцам?

Метод столбцов — это метод математических вычислений. Вычитание столбцов и добавление столбцов устанавливают число в столбцы в зависимости от разрядного значения каждой цифры.

Что означает столбец в математике?

В математике столбец — это расположение чисел друг над другом.

Как объяснить ребенку вычитание?

Вычитание можно объяснить как удаление или нахождение разницы. Например, «насколько больше 12, чем 10?».

Хотите знать, как объяснить своим детям другие математические слова? Ознакомьтесь с нашим словарем для начинающих по математике или попробуйте эти:

  • Что такое овладение математикой?
  • Обучение сложению и вычитанию KS2
  • Что такое сложение столбцов?

Вы можете найти множество рабочих листов для учеников начальных классов в Third Space Learning Math Hub.

Индивидуальные онлайн-уроки математики, которым доверяют школы и учителя
Каждую неделю репетиторы-специалисты по математике Third Space Learning помогают тысячам детей младшего школьного возраста проводить еженедельные индивидуальные онлайн-уроки и занятия по математике. С 2013 года мы помогли более 125 000 детей стать более уверенными в себе и способными к математике. Узнайте больше или запросите персональное предложение, чтобы рассказать нам о ваших потребностях и о том, как мы можем помочь.

Наша онлайн-программа обучения математике предоставляет каждому ребенку индивидуального профессионального репетитора по математике

Вычитание — определение, примеры | Вычитание в числовой строке

Вычитание — это процесс вычитания одного числа из другого. Это основная арифметическая операция, которая обозначается символом вычитания (-) и представляет собой метод вычисления разницы между двумя числами.

1. Что такое вычитание?
2. Формула вычитания
3. Как решить задачи на вычитание?
4. Вычитание с помощью числовой строки
5. Словесные задачи на вычитание из реальной жизни
6. ​​ Часто задаваемые вопросы о вычитании

Что такое вычитание?

Вычитание — это операция, используемая для нахождения разницы между числами. Когда у вас есть группа объектов и вы убираете из нее несколько объектов, группа становится меньше. Например, вы купили 9кексы на свой день рождения и ваши друзья съели 7 кексов. Теперь у вас осталось 2 кекса. Это можно записать в виде выражения вычитания: 9 — 7 = 2 и читается как «девять минус семь равно двум». Когда мы вычитаем 7 из 9, (9 — 7) мы получаем 2. Здесь мы выполнили операцию вычитания двух чисел 9 и 7, чтобы получить разницу 2.

Символ вычитания

В математике у нас есть разные символы. Символ вычитания является одним из важных математических символов, которые мы используем при выполнении вычитания. В предыдущем разделе мы читали о вычитании двух чисел 9и 7. Если мы наблюдаем это вычитание: (9 7 = 2), символ (-) соединяет два числа и завершает данное выражение. Этот символ также известен как знак минус.

Формула вычитания

Когда мы вычитаем два числа, мы используем некоторые термины, используемые в выражении вычитания:

  • Уменьшаемое: число, из которого вычитается другое число.
  • Вычитаемое: Число, которое нужно вычесть из уменьшаемого.
  • Разница: Конечный результат после вычитания вычитаемого из уменьшаемого.

Формула вычитания записывается как: Уменьшаемое — Вычитаемое = Разность

Давайте разберемся с формулой вычитания или математическим уравнением вычитания на примере.

Здесь 9 — уменьшаемое, 7 — вычитаемое, а 2 — разность.

Как решить задачи на вычитание?

При решении задач на вычитание однозначные числа можно вычитать простым способом, но для больших чисел мы разбиваем числа на столбцы, используя соответствующие разряды, например, единицы, десятки, сотни, тысячи и т. д. При решении таких задач мы можем столкнуться с некоторыми случаями с заимствованием и без заимствования. Вычитание с заимствованием также известно как вычитание с перегруппировкой. Когда уменьшаемое меньше вычитаемого, мы используем метод перегруппировки. При перегруппировке мы заимствуем 1 число из предыдущего столбца, чтобы уменьшаемое стало больше вычитаемого. Давайте разберемся в этом с помощью нескольких примеров.

Вычитание без перегруппировки

Пример: Вычтите 25632 из 48756.

Примечание. При вычитании мы всегда вычитаем меньшее число из большего, чтобы получить правильный ответ.

Решение: Выполните указанные шаги и попытайтесь связать их со следующим рисунком.

Шаг 1: Начните с разряда единиц. (6 — 2 = 4)
Шаг 2: Переход к разряду десятков. (5 — 3 = 2)
Шаг 3: Теперь вычтите цифры в разряде сотен. (7 — 6 = 1)
Шаг 4: Теперь вычтите разряд тысяч. (8 — 5 = 3)
Шаг 5: Наконец, вычтите цифры в десятитысячном разряде. (4 — 2 = 2)
Шаг 6: Таким образом, разница между двумя заданными числами составляет: 48756 — 25632 = 23124.

Вычитание с перегруппировкой

Пример: Вычтите 9002 из

    . Решение: Выполните указанные шаги и попытайтесь связать их со следующим рисунком.
    Нам нужно решить: 8162 — 3678
    Шаг 1: Начать вычитание цифр с единицы. Мы видим, что 8 больше 2. Итак, мы позаимствуем 1 из столбца десятков, что составит 12. Теперь 12 — 8 = 4 единицы.
    Шаг 2: После прибавления 1 к единицам на предыдущем шаге, 6 становится 5. Теперь давайте вычтем цифры в разряде десятков (5 — 7). Здесь 7 больше 5, поэтому мы возьмем 1 из столбца сотен. Получится 15. Итак, 15 — 7 = 8 десятков.
    Шаг 3: На шаге 2 мы поставили 1 в столбце десятков, поэтому у нас остался 0 в разряде сотен. Чтобы вычесть цифры на разряде сотен, то есть (0 — 6), мы возьмем 1 из столбца тысяч. Получится 10. Итак, 10 — 6 = 4 сотни.
    Шаг 4: Теперь давайте вычтем цифры в разряде тысяч. После присвоения 1 столбцу сотен мы имеем 7. Таким образом, 7 — 3 = 4
    Шаг 5: Таким образом, разница между двумя заданными числами составляет: 8162 — 3678 = 4484

    Вычитание с помощью числовой строки

    Числовая линия — это наглядное пособие, помогающее нам понять вычитание, поскольку оно позволяет нам переходить вперед и назад по каждому числу. Чтобы понять, как это работает, давайте рассмотрим вычитание с помощью числовой прямой. Давайте вычтем 4 из 9, используя числовую прямую. Начнем с того, что отметим цифру 9 на числовой прямой. Когда мы вычитаем, используя числовую прямую, мы считаем, перемещая одно число за раз влево. Так как мы вычитаем 4 из 9, мы будем двигаться 4 раза влево. Число, на которое вы приземлитесь после 4 прыжков назад, и есть ответ. Таким образом, 9 — 4 = 5.

    Словесные задачи на вычитание из реальной жизни

    Концепция вычитания часто используется в нашей повседневной деятельности. Давайте разберемся, как решать задачи на вычитание из реальной жизни с помощью интересного примера.

    Пример: На футбольном матче присутствовало 4535 зрителей. После первой подачи стадион покинули 2332 зрителя. Найдите количество оставшихся зрителей.

    Решение:
    Дано:
    Общее количество зрителей, присутствовавших в первом иннинге = 4535; Количество зрителей, покинувших стадион после первой подачи = 2332
    Здесь 4535 — уменьшаемое, а 2332 — вычитаемое.

    Чт Х Т О
    4 5 3 5
    -2 3 3 2
    2 2 0 3

    Следовательно, количество оставшихся зрителей = 2203.

    Важные примечания по вычитанию:

    Вот несколько важных замечаний, которым вы можете следовать при выполнении вычитания в повседневной жизни.

    • Любую задачу на вычитание можно преобразовать в задачу на сложение и наоборот.
    • Вычитание 0 из любого числа дает само число как разницу.
    • Когда из любого числа вычитается 1, разница равняется предшествующему числу.
    • Такие слова, как «Минус», «Меньше», «Разница», «Уменьшение», «Отнять» и «Вычесть», указывают на то, что вам нужно вычесть одно число из другого.

    Темы, связанные с вычитанием

    Ознакомьтесь с этими интересными статьями, чтобы узнать о вычитании и связанных с ним темах.

    • Двоичное вычитание
    • Калькулятор вычитания
    • Сложение и вычитание дробей
    • Вычитание комплексных чисел
    • Вычитание дробей

     

    Примеры вычитания

    1. Пример 1: г. В международном матче по крикету Шри-Ланка забила 236 пробежек, а Индия — 126 пробежек. Сколько еще ранов должна набрать Индия, чтобы сравняться с количеством ранов, набранных Шри-Ланкой?

      Решение:

      Раны, набранные Шри-Ланкой = 236; Раны, набранные Индией = 126
      Чтобы найти количество ранов, которое Индия должна набрать больше, чтобы оно равнялось количеству ранов, набранных Шри-Ланкой, мы вычтем 126 из 236.

      H T O
      2 3 6
      — 1 2 6
      1 1 0

      Таким образом, Индия должна набрать еще 110 очков, чтобы сравняться с числом очков Шри-Ланки.

    2. Пример 2: Джерри собрал 189 ракушек, а Ева собрала 54 ракушки. Кто собрал больше ракушек и на сколько?

      Решение:

      Количество снарядов, собранных Джерри = 189; Количество ракушек, собранных Евой = 54

      Это показывает, что Джерри собрал больше ракушек. Вычтем 189- 54, чтобы получить разницу.

      ГТО
      1 8 9
      — 0 5 4
      1 3 5

      Таким образом, Джерри собрал на 135 ракушек больше, чем Ева.

    3. Пример 3: Во время ежегодной охоты за пасхальными яйцами участники нашли в клубе 2469 яиц, из которых 54 пасхальных яйца были разбиты. Сможете ли вы узнать количество неразбитых яиц?

      Решение:

      Количество пасхальных яиц, найденных в клубе = 2469; Количество разбитых пасхальных яиц = 54; Общее количество неразбитых яиц =?

      Теперь мы вычтем количество разбитых яиц из общего количества яиц.

      Чт Х Т О
      2 4 6 9
      — 5 4
      2 4 1 5

      Таким образом, количество неразбитых яиц равно 2415.

    перейти к слайдуперейти к слайдуперейти к слайду

    Разбивайте сложные концепции с помощью простых визуальных средств.

    Математика больше не будет сложным предметом, особенно когда вы понимаете концепции с помощью визуализаций.

    Записаться на бесплатный пробный урок

    Практические вопросы по вычитанию

     

    перейти к слайдуперейти к слайду

    Часто задаваемые вопросы о вычитании

    Где мы используем вычитание?

    Вычитание используется в нашей повседневной жизни. Например, если мы хотим узнать, сколько денег мы потратили на купленные товары, или сколько денег осталось у нас, или если мы хотим подсчитать время, оставшееся до завершения задачи, мы используем вычитание.

    Какие существуют типы вычитания?

    Типы вычитания означают различные методы, используемые при вычитании. Например, вычитание с перегруппировкой и без нее, вычитание с использованием числовых таблиц, вычитание с использованием числовой прямой, вычитание небольших чисел с помощью пальцев и так далее.

    Что такое стратегии вычитания?

    Стратегии вычитания — это различные способы изучения вычитания. Например, с помощью числовой строки, с помощью таблицы значений разрядов, разделения десятков и единиц, а затем их вычитания по отдельности и многих других.

    Приведите несколько примеров на вычитание.

    Реальные примеры вычитания могут быть разными. Например, если у вас есть 5 яблок, а ваш друг съел 3 яблока. С помощью вычитания мы можем узнать количество оставшихся яблок: 5 — 3 = 2. Итак, у вас осталось 2 яблока. Аналогично, если в классе 16 учеников, из них 9 девочек, то мы можем узнать количество мальчиков в классе, вычитая 9 из 16. (16 — 9 = 7). Итак, мы знаем, что в классе 7 мальчиков.

    Какие три части вычитания?

    3 части вычитания называются следующим образом:

    • Уменьшаемое: число, из которого мы вычитаем другое число, называется уменьшаемым.
    • Вычитаемое: число, которое вычитается из уменьшаемого, называется вычитаемым.
    • Разница: Конечный результат, полученный после выполнения вычитания, известен как разница.

    Как написать вычитание?

    При записи вычитания двумя важными символами являются «-» (минус) и «=» (равно). Знак минус означает, что одно число вычитается из другого числа. И знак равенства дает окончательный результат.

    Скачать БЕСПЛАТНЫЕ учебные материалы

    Скачать бесплатно Рабочие листы для 1 класса

    Скачать бесплатно Рабочие листы для 2 класса

    Скачать бесплатно Рабочие листы для 3 класса

    Скачать бесплатно Рабочие листы для 4 класса

    Скачать бесплатно Рабочие листы KG

    0000 Вычитание — | Основы арифметики

    На этой странице рассказывается об основах арифметики, простейшем способе манипулирования числами посредством вычитания (-).

    См. другие наши арифметические страницы, для обсуждения и примеров: Сложение (+), Умножение ( × ) и Деление ( ÷ ).

    Вычитание

    Вычитание — это термин, используемый для описания того, как мы «вынимаем» одно или несколько чисел из другого.

    Вычитание также используется, чтобы найти разница между двумя числами. Вычитание противоположно сложению. Если вы еще этого не сделали, мы рекомендуем прочитать нашу страницу дополнений.

    Знак минус «-» используется для обозначения операции вычитания, например, 4 − 2 = 2. Знак «−» может использоваться несколько раз по мере необходимости: например, 8 − 2 − 2 = 4.

    Этот расчет правильный, но его можно упростить, сложив числа, которые мы вычитаем. В нашем примере 8 — 2 — 2 = 4 можно упростить до 8 — 4 = 4 (две двойки были сложены вместе, чтобы получить 4, которые затем вычитаются из 8).

    Предупреждение


    Необходимо соблюдать осторожность при использовании знака «-». Числа, имеющие отрицательное значение, записываются с предшествующим «-», поэтому минус два записывается как -2. Это просто означает, что на 2 меньше нуля или на 2 меньше нуля.

    Для получения дополнительной информации см. нашу страницу о положительных и отрицательных числах .

    Остерегайтесь знаков и порядка при вычитании

    Когда мы выполняем вычисление сложения , порядок сложения чисел не имеет значения.

    Например,
    8 + 3 + 5 совпадает с 3 + 8 + 5 и дает нам тот же ответ, 16. с порядком номеров.

    Обычно при вычитании мы сначала пишем число, которое мы вычитаем из , а числа, которые мы отнимаем, в любом порядке после этого.

    Например,
    8 − 5 = 3
    Это НЕ то же самое, что 5 − 8 = −3

    Мы видим, что у нас тот же числовой ответ (3), но его значение другое: 3 в первом расчете и минус 3 (−3) во втором.

    Точно так же 8 − 5 − 3 = 0, но 5 − 8 − 3 = −6, что является совершенно другим ответом.

    Причина, по которой ответы различаются, не в том, что мы расставили числа в «неправильном» порядке, а в том, что мы не позаботились о том, чтобы отметить, положительные они или отрицательные.

    В нашем примере 8 — положительное число, поэтому мы могли бы записать его как «+ 8», и это было бы правильно, но по соглашению нам не нужно писать символ «+». Однако символ «+» очень важен, если мы изменим порядок, как и символы «-», предшествующие 5 и 3.

    Вот последний пример, переписанный так, чтобы дать правильный ответ:

    8 − 5 − 3 = 0, как и раньше, и − 5 + 8 − 3 = 0, что дает тот же ответ. В этом случае мы записали числа в том же порядке, что и раньше, но приняли во внимание их положительное или отрицательное значение.

    Более подробное объяснение и примеры см. в разделе «Вычитание в особых ситуациях: нулевые и отрицательные числа » ниже.

    Вычитание

    Простое вычитание можно выполнять так же, как и сложение, путем подсчета или использования числовой строки:

    Если у Фиби 9 конфет, а у Люка 5 конфет, в чем разница?

    Начиная с меньшего числа (5) и считая до большего числа (9).

    6 (1), 7 (2), 8 (3), 9 (4).

    У Фиби на 4 конфеты больше, чем у Люка, разница в конфетах 4.

    Итак: 9 − 5 = 4 .

    Для более сложного вычитания, когда использование подсчета нецелесообразно, полезно записывать наши числа в столбцах одно над другим — аналогично вычислению сложения.

    Предположим, Майк зарабатывает 755 фунтов стерлингов в неделю и платит 180 фунтов стерлингов в неделю за аренду. Сколько денег осталось у Майка после того, как он заплатил за квартиру?

    В этом примере мы собираемся вычесть 180 фунтов стерлингов из 755 фунтов стерлингов. Сначала мы пишем начальный номер, а под ним номер, который мы убираем, следя за тем, чтобы числа были в правильных столбцах.

    Сотни Десятки Единиц
    7 5 5
    1 8 0

    Шаг 1: Сначала мы выполняем вычитание чисел в столбце «Единицы» справа, затем записываем ответ внизу в том же столбце. В этом случае 5 − 0 = 5,

      Сотни Десятки Единиц
      7 5 5
      1 8 0
    Итого     5

    Шаг 2: Используя тот же подход, что и при вычислении сложения, мы работаем по столбцам справа налево. Далее нам нужно вычесть числа в столбце десятков. В нашем примере нам нужно вычесть восемь из пяти (5 − 8), но 8 больше 5, поэтому мы не можем этого сделать, так как в итоге получим отрицательное число. Нам нужно позаимствовать число из столбца сотен. Это может быть сложной концепцией, и мы рассмотрим ее более подробно ниже: у нас есть 7 в столбце сотен, поэтому мы «заимствуем» 1 для столбца десятков, оставляя нам 6 в сотнях. Перечеркните 7 и напишите 6 в колонке сотен, чтобы потом не ошибиться. Переместите 1 в столбец десятков и напишите его перед 5. Мы не добавляем «1» к десяткам, мы кредитуем «1 лот из 10». Итак, вместо 5 десятков у нас теперь 15 десятков.

    15 больше восьми, поэтому мы можем выполнить вычитание в столбце десятков. Возьмите 8 из 15 и запишите ответ (7) внизу столбца десятков.

    Сотни Десятки Единиц
    7 6 15 5
    1 8 0
    Итого 7 5

    Шаг 3:  Наконец, отнимите 1 от 6 в столбце сотен. 6 − 1 = 5, поэтому поставьте 5 в ответе столбца сотен, чтобы получить окончательный ответ. У Майка осталось 575 фунтов стерлингов после того, как он заплатил арендную плату.

      Сотни Десятки Единиц
      7 6 15 5
      1 8 0
    Итого 5 7 5



    Заимствование при вычитании

    Заимствование , как в приведенном выше примере, может привести к путанице при вычислениях с вычитанием. Это похоже на «перенос» в вычислениях сложения, но в обратном порядке, потому что вычитание является обратным (противоположным) сложению.

    Повторное заимствование может происходить при вычислении вычитания.
    Предположим, у нас есть 10,01 фунта стерлингов, и мы хотим забрать 9,99 фунта стерлингов. Мы можем решить это, не записывая ничего — ответ 0,02 фунта стерлингов или 2 пенса. Однако если мы запишем этот расчет формально, то понятие заимствования станет более ясным.

    В этом примере мы проигнорировали десятичную точку и записали числа как 1001 и 999.

    1 0 0 1
    9 9 9

    Начиная со столбца единиц измерения справа, нам нужно отнять 9 от 1. В наших вычислениях с вычитанием действует правило (как в приведенном выше примере): мы никогда не отнимаем большее число от меньшего числа, потому что оно даст нам отрицательный ответ.

    Чтобы расчет работал, нам нужно заимствовать число из следующего столбца слева. В столбце десятков стоит 0, поэтому заимствовать нечего, поэтому нам нужно перейти к следующему столбцу слева. Столбец сотен также имеет 0, поэтому мы также не можем заимствовать из этого столбца, поэтому переходим к следующему столбцу слева. В столбце тысяч есть 1, поэтому мы можем позаимствовать его и переместить в следующий столбец справа, столбец сотен. Мы перечеркиваем 1 в столбце тысяч, чтобы избежать ошибок позже.

    Одна тысяча равна 10 сотням, так что теперь у нас есть 10 в столбце сотен, где раньше был ноль:

    Перенесено 0 10
    1 0 0 1
      9 9 9

    Однако это не помогает с 1 − 9 (в столбце единиц), потому что у нас все еще есть ноль для заимствования в столбце десятков, но это первый шаг в процессе.

    Теперь, когда у нас есть 10 сотен, мы можем позаимствовать одну из них для столбца десятков. Сотня — это то же самое, что 10 десятков, поэтому мы переносим 10 в столбец десятков. Мы не должны забывать корректировать столбец сотен, поэтому мы перечеркиваем 10 и вместо этого пишем 9.

    Перевозится 9 10
    Перевозится 0 10
    1 0 0 1
      9 9 9

    Наконец, мы можем выполнить вычитание в столбце единиц, позаимствовав 1 десяток из столбца десятков. Это оставляет 9 десятков в столбце десятков и 10 + 1, которые у нас уже были в столбце единиц, что дает нам 11 единиц.

    Перевозится 9 10
    Перевозится 9 10
    Перевозится 0 10
    1 0 0 1
      9 9 9

    Теперь мы можем выполнить полный расчет, начиная со столбца единиц, 10 + 1 = 11 − 9.= 2. Тогда в столбце десятков 9 − 9 = 0. То же самое для столбца сотен 9 − 9 = 0. Наконец, в столбце тысяч 0 − 0 = 0.

    Перенесено 9 10
    Перевозится 9 10
    Перевозится 0 10
    1 0 0 1
      9 9 9
    Итого 0 0 0 2

    Одолжив несколько раз, мы пришли к ответу 2. Когда мы заменяем десятичную точку, у нас есть 0,02 фунта стерлингов.


    Вычитание в особых случаях: нули и отрицательные числа

    Если бы мы выполняли простое сложение, мы могли бы считать в уме или, возможно, на пальцах. Когда мы выполняем вычитание, особенно если оно включает отрицательные числа, полезно представить, что мы идем по прямой. Каждый шаг — это число в этой строке. Если мы начнем с нуля, каждый шаг вперед будет прибавлять число, а каждый шаг назад отнимать единицу. Самое важное, что нужно помнить, это то, что мы всегда смотрим в позитивном направлении. Возможно, вам будет полезно представить свою линию как восхождение вверх и вниз по лестнице, где каждая ступенька представляет собой число. Или, возможно, вы более знакомы с путешествием вверх и вниз по высотному блоку в лифте, где ноль — это первый этаж, положительные числа — над землей, а отрицательные числа — в подвале.

    Если бы мы нарисовали эту линию на листе бумаги, она выглядела бы как линейка. Мы можем двигать пером вперед и назад по линии так же, как представляем себе свои шаги назад и вперед. Это называется числовой линией и является очень полезным инструментом для сложения и вычитания.

    Мы воспользуемся этой аналогией, чтобы понять следующие примеры.

    Когда числа одинакового значения вычитаются друг из друга, результат всегда равен нулю: 19−19 = 0,

    Используя нашу аналогию, начав с нуля, если мы пройдем 19 шагов вперед по линии, а затем 19 шагов назад, мы вернемся к нулю.

    При вычитании нуля из любого числа число остается неизменным: 19−0 = 19.

    Используя нашу числовую прямую, мы начинаем с 19 и идем назад 19.

    Когда мы вычитаем любое положительное число из нуля, ответ равен отрицательное : 0 – 15 = –15

    Помните из наших предыдущих примеров, что положительное число обычно не нужно записывать с положительным знаком. Когда мы видим число «67», математическое соглашение говорит нам, что оно положительное, то есть «+67».

    В этом примере из нуля вычитаем +15: 0 – (+15) = –15. Используя нашу аналогию, мы начинаем с нуля и делаем 15 шагов назад.

    Когда мы вычитаем любое положительное число из отрицательного числа , ответ становится равным 9.0279 более отрицательное .

    Например, если мы начнем с нашего ответа сверху (–15) ​​и вычтем 6, мы получим: –15 – 6 = –21. Помните, что «6» положительно, поэтому мы могли бы записать –15 – (+6) = –21, и это означает то же самое. Используя нашу числовую линию, чтобы помочь нам понять, мы начинаем с -15. Мы делаем шесть шагов назад, по-прежнему глядя в позитивном направлении. Мы заканчиваем на 21 шаге назад от нуля, то есть –21.

    Но что произойдет, если нам нужно вычесть отрицательное число из любого другого числа?

    Начнем с примера: 15 – (–6) = 15 + 6 = 21

    Правило два минуса дают положительное , то есть вычитание отрицательного числа становится сложением.

    Давайте вернемся к нашей числовой прямой, чтобы нам было легче понять: начиная с 15, мы знаем, что нам нужно двигаться назад (в отрицательном направлении), потому что мы делаем вычитание. Но у нас есть отрицательное число для вычитания, поэтому, чтобы проиллюстрировать это, мы должны перевернуть . Затем мы перемещаемся назад на 6 позиций, чтобы получить ответ. Поворачиваясь, а затем двигаясь назад (два отрицательных числа), наше общее направление движения находится в положительном направлении , то есть мы выполнили сложение .

    Вычитание отрицательного числа — это абстрактная концепция, и вы можете подумать, что в повседневной жизни она не используется. Ведь мы не можем удержать отрицательное количество яблок или налить отрицательное количество кофе. Однако это очень важно, когда речь идет о таких математических понятиях, как векторов . Вектор имеет направление , а также величину , поэтому, например, важно не только то, как далеко проплыла лодка, но нам также нужно знать направление, в котором она путешествовала.



    Дополнительное чтение из книги «Навыки, которые вам нужны»


    Основы счета
    Часть руководства «Навыки, которые вам нужны» для счета

    операции и начать манипулировать числами. Он также включает примеры из реальной жизни, чтобы прояснить, как эти концепции полезны в реальной жизни.

    Если вы хотите освежить свои знания или помочь своим детям в обучении, эта книга для вас.


    23 занимательных занятия по вычитанию, которые понравятся детям и учителям

    По некоторым причинам вычитание часто дается учащимся немного сложнее, чем сложение. Вот почему хорошо иметь в своем наборе инструментов целый набор умных действий по вычитанию. Это одни из наших любимых упражнений для занятий в школе или дома.

    Напоминаем, что WeAreTeachers может собирать часть продаж по ссылкам на этой странице. Спасибо за Вашу поддержку!

    1. Начните с якорной диаграммы.

    Привязочные диаграммы дают детям информацию, на которую можно ссылаться, когда они работают над новыми концепциями и навыками. Нам нравится эта якорная диаграмма вычитания за все детали, которые она предоставляет.

    Подробнее: Lucky Little Learners 

    2. Прочтите книгу по вычитанию.

    Чтение вслух — это интересный способ познакомить детей с новыми математическими понятиями. Есть много отличных книг, которые помогут вам в занятиях по вычитанию, например, наша многолетняя любимица 9.0279 Кот Пит и его четыре крутые пуговицы .

    Подробнее: Щепотка Kinder

    3. Сосчитайте пуговицы Пита.

    Если вашим ученикам понравилась книга, попробуйте эту идею, которая включает в себя настоящие кнопки для практики вычитания. Так весело!

    Узнать больше: Inspiration Laboratories

    4. Залезть на обезьянье дерево.

    Five Little Monkeys — еще одна любимая книга, в которой математические понятия встроены в историю. Построй собственное дерево, по которому обезьяны могли бы лазить, или используй кукольную кровать для прыжков!

    Дополнительная информация: Детский сад миссис Рикки

    5.

    Предлагайте различные стратегии.

    Дайте детям множество стратегий для понимания вычитания. Они могут использовать их при работе над вычитанием и упражнениями.

    Подробнее: Горжусь тем, что учусь на первом курсе

    6. Разбейте пластилин Play-Doh.

    Дети почему-то просто обожают разбивать вещи! Скатывайте шарики из пластилина Play-Doh и давайте детям гладить их, пока они вычитают. (Другие способы использования пластилина Play-Doh для обучения вы найдете здесь.)

    Подробнее: Мама Папа Бабба

    7. Ударьте по мячу, чтобы вычесть.

    Вот еще одно упражнение на вычитание, которое позволяет детям сжечь немного энергии! Сделайте трехмерную рамку для десяти, прорезав отверстия в обувной коробке. Положите шарики для пинг-понга в отверстия, затем ударьте по ним игрушечным молотком, чтобы вычесть их.

    Подробнее: Планирование игры

    8. Накорми голодного пингвина.

    Этот голодный пингвин поможет вашим ученикам научиться вычитать! Накормите его бумажными рыбками и напишите уравнения.

    Подробнее: Натали Линн Детский сад

    9. Визуализируйте проблему.

    Упражнения на вычитание, такие как задачи на вычитание, легче выполнять, когда дети могут визуализировать действие. Попробуйте нарисовать картинки или разыграть историю, чтобы воплотить идею в жизнь.

    Подробнее: Susan Jones Teaching

    10. Сортируйте карточки.

    Пометьте ряды бумажных пакетов цифрами от 1 до 10, затем рассортируйте карточки по нужным пакетам. Сделайте это упражнение на время, и две команды соревнуются, чтобы узнать, кто быстрее (и точнее)!

    Узнать больше: Шведский стол в детском саду

    11. Чаша и вычитание.

    Это одно из самых популярных упражнений на вычитание. Дети шарят, сбивают булавки и вычитают их из общего количества. Это слишком весело, чтобы учиться, не так ли?

    Подробнее: Щепотка Kinder

    12. Устройте гонку на вычитание LEGO.

    Каждый игрок начинает игру со стопкой кубиков LEGO. Бросьте кубик и уберите выпавшее число, каждый раз произнося уравнение вслух. Первый до нуля побед!

    Подробнее: The Kindergarten Connection

    13. Найдите ключ от замка.

    Приобретите набор замков, которые можно использовать для самых разных целей, в том числе для операций вычитания. Эти 8 упаковок от Amazon бывают разных цветов, и их должно хватить на долгие годы.

    Узнать больше: Детский сад дифференцированного типа

    14. Используйте кубики LEGO для двузначных чисел.

    Готовы поговорить о «заимствовании» и двузначной математике? Упростите концепцию, используя кубики LEGO.

    Подробнее: Реальная жизнь дома

    15. Возьмите обруч.

    Начните с подбрасывания мешков с фасолью и посмотрите, сколько из них вы сможете попасть в обруч. Затем превратите результаты в уравнения вычитания.

    Подробнее: Маленькие ученики мисс Джейкоб

    16. Спросите: «Сколько я прячу?»

    Упражнения на вычитание, подобные этому, — интересный способ познакомить маленьких учеников с этой идеей. Начните с того, что положите на стол несколько небольших предметов. Затем накройте часть рукой. Затем спросите, сколько осталось и сколько вы прячете. Простой, но эффективный.

    Узнать больше: PreKinders

    17. Стопка чашек.

    Детям очень нравится ставить стаканчики друг на друга. Включите в смесь практику вычитания, пометив чашки уравнениями и решениями, а затем предложите учащимся сложить чашки друг с другом, чтобы показать правильные ответы.

    Подробнее: Шведский стол в детском саду

    18. Решайте и играйте в Бинго.

    Сначала дети разрабатывают правильные ответы на задачи на вычитание из двух цифр в этой бесплатной распечатке. Затем они вырезают их и наклеивают на доску для бинго. Наконец, сыграйте вместе в бинго!

    Подробнее: Начальная школа Giggles

    19. Бросьте несколько мешков с фасолью.

    Напишите сетку случайных чисел на тротуаре (или приклейте одну на полу в классе). Ученик бросает два мешочка с фасолью на числа, а затем составляет уравнение вычитания из полученных результатов.

    Подробнее: Education.com

    20. Используйте сумку-слайдер на молнии.

    Номер пакета на молнии? Гений! Эта умная идея — полезный инструмент для многих действий по вычитанию. (Ознакомьтесь с другими идеями числовых линий здесь.)

    Подробнее: Миссис Т. Первый класс

    21. Наденьте бусины на ершики для труб.

    Сделайте несколько удобных математических манипуляций с ершиками для труб и бусинками. Сдвиньте бусины, чтобы увидеть уравнения вычитания в действии!

    Подробнее: Советы и идеи от Анны Гарднер

    22.

    Соберите шар-разрушитель.

    Еще один шанс устроить небольшой хаос! Подобно игре в боулинг, описанной выше, дети складывают несколько блоков, затем сбивают их и смотрят, сколько осталось. Шумно, но весело!

    Подробнее: Math Geek Mama

    23. Выучите рифмы на вычитание.

    Эти стишки помогут детям, работающим над более сложными упражнениями на вычитание, помочь им вспомнить, когда им нужно «заимствовать» из следующего столбца разрядного значения, а когда нет.

    Узнать больше: Эми Лемонс

    Ищете другие элементарные математические идеи? Ознакомьтесь с этими упражнениями для обучения сложению, умножению и делению.

    Плюс, более 60 потрясающих веб-сайтов для преподавания и изучения математики.

    Обучение вычитанию младших школьников: объяснение лучших стратегий

    • Дом
    • >
    • Блог
    • >
    • Как научить маленьких детей вычитанию: четыре проверенных метода

    Джессика Камински

    10 минут чтения

    16 февраля 2022 г.

    Большинство взрослых могут вычитать числа за несколько секунд, хотя это сложнее, чем складывать. Иногда вы можете мгновенно решить 10 — 9 или 14 — 6, потому что выучили факты вычитания еще в школе.

    Тем временем маленькие дети стараются усвоить основы. Как научить детей правильно считать двузначные числа в уме? Вы должны построить математическую основу.

    На этот раз мы объясним, как учить маленьких детей вычитанию и каких стратегий обучения следует избегать, так что продолжайте читать.

    Когда детям следует изучать вычитание?

    Вычитание — одно из первых математических понятий, которые дети начинают изучать вместе со сложением. В школах учат вычитанию постепенно. Обычно это происходит уже в дошкольном возрасте. В это время педагоги знакомят дошкольников с прибылями и убытками и математическими операциями, их отражающими.

    Дети учатся связывать числа с предметами и понимают, что могут вычислять и выражать потери с помощью математического языка.

    Несмотря на то, что общие базовые стандарты делают все возможное, чтобы не перегружать учащихся сложной теорией вычитания сразу, дети все равно испытывают трудности с этой операцией.

    Вы можете помочь своему ребенку научиться вычитанию, применяя простые стратегии обучения. Продолжайте читать, чтобы узнать больше о вычитании и четырех стратегиях, чтобы научить этому дошкольников и первоклассников.

    Просмотрите рабочие листы для печати на вычитание для ваших детей!

    Что такое вычитание фактов?

    Возможно, вы слышали о фактах вычитания — это уравнения вычитания, включающие определенные комбинации чисел в пределах десятков, например 10 — 5 = 5.

    Считайте их общеизвестными, например, что Земля вращается вокруг Солнца. Вот несколько примеров:

    • Вычитание нуля из числа не меняет его значения: 5 – 0 = 5; 9 – 0 = 9; 3 – 0 = 3 и так далее.
    • Простые уравнения на вычитание, например: 10 – 3 = 7; 8 – 4 = 4; 14 – 6 = 8 и так далее.

    Зачем вычитать факты? Это потому, что их запоминание помогает вам вычитать числа в уме, не думая — вы сразу знаете ответ, когда видите 10 — 5 или 6 — 3.

    Свободное владение фактами вычитания позволяет намного быстрее вычислять в уме двузначные и трехзначные числа.

    Тем временем дети в начальной школе не научились вычитанию до такой степени, чтобы знать эти факты наизусть.

    1:1 Уроки математики

    Хотите воспитать гения?
    Начните изучать математику с Brighterly Давайте начнем изучать математику!

    Четыре стратегии обучения вычитанию для начинающих

    Давайте рассмотрим четыре стратегии, чтобы объяснить вычитание вашему дошкольнику и первокласснику.

    Обучение вычитанию: первый класс и детский сад

    Вот как работает обучение вычитанию в детском саду:

    • Положите на стол перед ребенком несколько яблок.
    • Сосчитай их: раз, два, три, четыре.
    • Объясните, что мы можем представить яблоки в виде чисел – четыре яблока равны 4.
    • Убери два яблока.
    • Еще раз пересчитайте оставшиеся фрукты: раз, два.
    • Из четырех мы убрали два, осталось два.
    • Теперь представим, что яблоки пронумерованы: 4 – 2 = 2.

    Вместо этого вы можете использовать детали Lego. Например, построить башню из лего высотой в восемь кирпичей. Уберите три кирпича, и он станет высотой всего в пять кирпичей.

    В результате ваш дошкольник может выполнять следующие действия по вычитанию чисел:

    • Считать предметы в группе.
    • Заберите предметы из группы.
    • Подсчитайте, что осталось в группе.

    Используя этот пример, вы можете объяснить, что вычитание равно отнятию.

    Практикуйте вычитание и знаки

    Как только ваши дети начнут ассоциировать вычитание с отнятием, им следует выучить словарь вычитания, чтобы переключиться на вычисления в уме.

    Вот термины вычитания:

    • Уменьшаемое – это число, от которого что-то отнимают.
    • Вычитаемое – это число, которое вы отнимаете от уменьшаемого.
    • Разность – это результат вычитания.

    Для 5 – 2 = 3 пять – уменьшаемое, два – вычитаемое, а три – разность.

    Математики используют термин «минус» и связанный с ним знак «-» для обозначения действия удаления.

    С точки зрения математики, мы должны произносить 5 – 2 = 3 как «пять минус два равно трем». Объясните эту теорию своему ребенку.

    Чем больше ваши дети упражняются в этих терминах, тем более бегло они осваивают вычитание.

    Обучение вычитанию в первоклассниках с помощью числовых линий

    Числовые линии помогают первоклассникам освоить вычитание в пределах 20. Вот как это работает:

    • Нарисуйте линию и разделите ее на 20 равных сегментов. Пронумеруйте их от 1 до 20.
    • Объясните, что ваш ребенок может вычитать, считая в обратном порядке по числовой прямой.

    Возьмем, к примеру, 8 – 5 = 3. Найдите 8 на числовой прямой. Затем прыгните пять раз назад: 8 → 7 → 6 → 5 → 4 → 3. Следовательно, 8 – 5 = 3.

    Помните, что числовые ряды подходят дошкольникам и первоклассникам, так как их математическая программа включает в себя вычитание с числами до 20.

    Вычитание любых чисел после 20 с числовыми линиями запутает вашего ребенка.

    Используйте игры на вычитание

    Исследования показывают, что повторяющиеся математические упражнения дают наилучшие результаты — помогают детям запоминать математические понятия и развивать беглость вычислений.

    Однако проблема в том, что ученики не выдерживают решения изнурительных математических упражнений по паре часов три раза в неделю. Хорошей новостью является то, что вы можете дать им достаточно математической практики с помощью веселых математических игр на вычитание. Забавные персонажи, плавная анимация и встроенные математические упражнения сделают изучение математики веселым и легким.

    Обучение вычитанию, которое убивает мотивацию к обучению

    Вы можете наткнуться на контрпродуктивные стратегии обучения вычитанию, которые кажутся хорошо написанными и простыми. Проверьте их ниже.

    Никогда не перекармливайте ребенка лишней информацией

    Многие родители (и некоторые учителя) попадают в ловушку, обучая вычитанию: они считают, что чем больше теории и инструментов они дадут, тем лучше их дети будут вычитать. Это не всегда так.

    Если вы объясните все о вычитании за один присест, ваш ребенок потеряется и запутается. Чтобы научить чему-то сложному, вы должны в первую очередь построить прочную основу.

    Математика кажется детям иностранным языком. Это противоречит здравому смыслу и иногда сводит их с ума (особенно вычитание с перегруппировкой). Таким образом, вы должны предоставить своему ученику математические знания, основанные на их текущем понимании.

    Не учите традиционному вычитанию с заимствованием

    Все мы знаем метод вычитания столбцов. Учителя начинают вводить его в третьем классе.

    Но многие преподаватели считают, что этот метод изначально ошибочен. Допустим, вам нужно решить 22 – 9. Вы пишете 23 в верхнем столбце и 9 в нижнем столбце.

    Затем вы вычитаете 9 из 3, что дает отрицательное число. А вот отрицательные числа дети учат в шестом классе. Здесь уже есть противоречие – учащиеся должны освоить понятие, которое требует знания, доступ к которому они получат только в будущем.

    Таким образом учителя ставят телегу впереди лошади. Вот почему репетиторы должны говорить третьеклассникам, что они не могут вычесть 9 из 3. Вместо этого они должны «одолжить» 1 из 2.

    В результате 2 становится 1, а 3 становится 13. Став взрослым, вы знаете, что это происходит, потому что вы перегруппировываете 23 в 10 и 13.

    Но это еще один недостаток – дети путаются, потому что у них нет твердого знания о местоположении.

    Когда дети вычеркивают 2 и 3 и пишут сверху 1 и 13, они не получают 10 и 13. Они получают 113, и они понятия не имеют, почему 113 – 9= 14.

    Наконец, вычитание с перегруппировкой не позволяет детям понять разрядные значения, поскольку оно подчеркивает единицы. Он учит детей вычитать единицы из единиц, не получая полной картины.

    Вопрос в том, почему учителя должны заставлять детей вычитать большие числа из меньших, вычеркивать цифры, записывать новые и перетасовывать разряды?

    Столько ненужной умственной работы. Почему же тогда мы должны смущать наших детей и наказывать их за наши ошибочные методы вычитания? Мы не должны, поэтому проверьте, как правильно научить первоклассников вычитанию, ниже.

    Как объяснить вычитание без заимствования?

    Учащиеся могут свободно вычитать без заимствования, если вы научите их разрядным значениям и познакомите с отрицательными числами. Первоклассникам и второклассникам не нужно знать все об отрицательных числах и их применении.

    Скажите им, что они могут получить отрицательную разницу.

    Они могут вычитать без заимствования, разбивая числа на основе их разрядности.

    Таким образом, ваш ребенок тоже перегруппирует значения мест. Но они делают это в расширенной форме, которая лучше иллюстрирует то, что происходит, чем вычитание столбцов с перегруппировкой:

    1. Разбить 23 на десятки и единицы: 10 (один-десять) и 13 (13 единиц).
    2. Вычесть единицы: 13 – 9 = 4.
    3. Объединить единицы и десятки: 10 + 4 = 14.

    Репетиторы Brighterly учат вашего ребенка вычитать

    Если ваш ребенок испытывает затруднения с вычитанием, отрицательными числами или перегруппировкой значений разрядов, вы можете поручить свои усилия по обучению профессионалам Brighterly.

    В рамках наших математических курсов для детей мы обучаем эффективным методам вычитания. Все, что вам нужно сделать, это указать класс вашего ребенка и знания математики. После этого вы можете записать своего ребенка на один из наших пакетов уроков математики. Следовательно, ваш ученик будет практиковать свои вычислительные навыки, изучать математические концепции и улучшать свои знания математических операций под нашим любезным наблюдением.

    Практический результат

    Научиться вычитанию труднее, чем сложению, но дети могут делать это без усилий, если учить этому правильно – начинать с малого и медленно. Пока вы создаете прочную основу, вы можете перейти к более сложным темам вычитания. Для этого вы можете объяснить вычитание как удаление, использовать числовые линии и попрактиковаться в словарном запасе вычитания.

    Если вы чувствуете, что преподавание математики утомляет вас, вы можете поручить эту задачу опытным преподавателям в любое время.

    Джессика Камински

    Джессика — опытный репетитор по математике с десятилетним опытом работы в этой области. Имея степень бакалавра и магистра математики, ей нравится воспитывать математических гениев, независимо от их возраста, уровня и навыков. Помимо репетиторства, Джессика ведет блог в Brighterly. У нее также есть опыт работы в области детской психологии, домашнего обучения и консультирования по учебным программам для школ и веб-сайтов EdTech.

    Что такое вычитание столбцов?

    Вычитание — это просто удаление одного числа из другого. Когда цифры простые, мы можем вычесть их в уме. Однако, когда числа становятся немного сложнее, тогда вычитание столбцов может быть эффективным и точным методом. При этом он работает правильно только в том случае, если вы сначала понимаете значение разряда, в противном случае слишком легко случайно перепутать числа!

     

     

     

    Давайте воспользуемся методом вычитания столбцов для вычисления 55 — 34.

     

      Десятки Единицы
      5 5
    3 4
         

     

    Всегда помещайте наибольшее число в верхнюю строку.

    Убедитесь, что даже если вы не маркируете его, у вас есть столбец «десятки» и столбец «единицы». Выстройте десятки прямо друг под другом и сделайте то же самое с единицами.

     

    1. Начните с единиц (или крайнего правого столбца, если у вас есть десятичные дроби).

    Всегда вычитайте нижние числа из верхних, даже если верхнее число выглядит меньше (подробнее об этом позже).

     

      Десятки Единицы
      5 5
    3 4
        1

     

    5 вычтите 4 из 1.

     

    1. Повторите с десятками (а затем с сотнями, тысячами и так далее, если они у вас есть).

     

      Десятки Единицы
      5 5
    3 4
      2 1

     

    5 вычесть 3 равно 2.

    (На самом деле это 50-30=20, но в столбце десятков нам не нужно беспокоиться об использовании нуля в качестве заполнителя).

    Ответ 21 теперь отображается внизу.

     

     

    Если верхняя цифра в любом из столбцов меньше, чем нижняя цифра, то мы должны занять.

     

    Совет. Нельзя просто поменять местами числа и вычесть верхнее из нижнего. Многие люди пытаются это сделать, но это не даст вам правильного ответа!

     

    Разработаем 132 — 64.

     

    1. Расставьте числа следующим образом;

     

      Сотни Десятки Единиц
      1 3 2
      6 4
           

     

    Убедитесь, что даже если вы не пометите их, у вас есть столбец «сотни», столбец «десятки» и столбец «единицы». Выстройте десятки прямо друг под другом и сделайте то же самое с единицами.

     

    Совет. Если в вычитаемом числе меньше цифр (как в нашем примере), следите за тем, чтобы десятки всегда располагались под десятками, а единицы — под единицами. Оставьте пробел или поставьте ноль, чтобы убедиться, что вы случайно не переместите свои цифры в неправильные столбцы.

     

    1. Начните с единиц (или крайнего правого столбца, если у вас есть десятичные дроби).

    Всегда вычитайте нижние числа из верхних, даже если верхние числа кажутся меньшими. В этом случае нам нужно отнять 4 от 2. Как?! Вы можете спросить.

    Мы делаем это, одалживая.

    В числе 132 3 десятка или 30. Мы просто «заимствуем» один из десятков. 30 становится 20, а затем к 2 единицам прибавляется еще десять, и получается 12.

    Теперь наш расчет становится таким: 12 – 4 = 8.

     

      Сотни Десятки Единиц
      1 2 3 12
      6 4
          8

     

    1. Повторить с десятками (или со следующим столбцом слева).

    Нам снова придется брать взаймы, потому что из 20 нельзя отнять 60.

    Мы просто «заимствуем» сотню, в результате чего у нас останется ноль в столбце сотен и 120 в столбце десятков.

    Наш расчет теперь становится 120 – 60 = 60

    (поскольку мы вычисляем в столбцах, нам не нужно показывать 0 из 120, так что это выглядит как 12, но на самом деле 120)

     

    Сотни Десятки Единиц
      1 12 3 12
      6 4
        6 8

     

    Ответ 68 теперь отображается внизу.

     

    У нас есть множество отличных рабочих листов, чтобы научить вас вычитанию столбцов. Мы перечислили некоторые из наиболее подходящих здесь, но просмотрите наш веб-сайт или выполните поиск по запросу «вычесть», чтобы найти больше ресурсов.

admin

Добавить комментарий

Ваш адрес email не будет опубликован. Обязательные поля помечены *